EPPP: Misc Practice Questions

अब Quizwiz के साथ अपने होमवर्क और परीक्षाओं को एस करें!

A White adult in Helms's __________ stage of White identity development recognizes his/her own racist views and justifies them by adopting beliefs that support White superiority and minority inferiority. A. reintegration B. disintegration C. pseudoindependence D. immersion-emersion

A

A picture of a knife is projected onto a screen so that it's briefly presented only to the right visual field of a split-brain patient, and a picture of a fork is then briefly presented only to the patient's left visual field. When asked to verbally identify what he has seen, the patient: A. will be able to say "knife" and, although he can't say "fork," will be able to pick out a fork with his left hand. B. will be able to say "fork" and, although he can't say "knife," will be able to pick out a knife with his right hand. C. will be able to say "knife" but won't be able to say "fork" or pick out a fork with his right or left hand. D. will be able to say "fork" but won't be able to say "knife" or pick out a knife with his right or left hand.

A

A progressive loss of dopamine-producing cells in the __________ has been linked to Parkinson's disease. A. substantia nigra B. reticular formation C. suprachiasmatic nucleus D. amygdala

A

A researcher conducts a study to compare four different diet programs on weight loss for individuals who have either high, moderate, or low self-esteem. The statistical analysis of the data he collects indicates that there are statistically significant main and interaction effects. This means that: A. interpreting the main effects without considering the interaction can lead to erroneous conclusions. B. interpreting the interaction is unnecessary because main effects are always most important. C. the interaction must be interpreted with caution because the main effects are significant. D. there must have been a calculation error since it's impossible to have both significant main and interaction effects.

A

Abramson, Metalsky, and Alloy's (1989) hopelessness model of depression proposes that hopelessness is a proximal cause of depression and that a sense of hopelessness is the result of: A. a lack of response contingent reinforcement. B. stable and global attributions for negative life events. C. depressogenic cognitive schemata. D. deficits in self-monitoring, self-evaluation, and self-reinforcement.

A

According to Carl Rogers (1951), incongruence between self and experience produces anxiety, which a person may respond to defensively by: A. denying or distorting the experience. B. adopting an exaggerated sense of self-worth. C. becoming fixated at the previous stage of development. D. developing a mistaken style of life.

A

According to Donald Super, the primary determinant of a person's occupational choice is which of the following? A. self-concept B. dominant interest(s) C. life experience D. prepotent needs

A

According to self-verification theory (Swann, Pelham, & Krull, 1989), people who are insecure about their physical appearance and have other negative self-views are most likely to be drawn to potential romantic partners who: A. verify their negative self-views. B. challenge their negative self-views. C. avoid making any positive or negative comments about them. D. talk mostly about themselves.

A

According to some investigators, the acquisition of most aspects of human development have a ________ period. This means that these skills can be acquired most easily during a particular period of time but can be acquired later, although it may be more difficult to do so. A. sensitive B. critical C. relative refractory D. absolute refractory

A

As described by Erikson, the positive outcome ("virtue") of the ego integrity versus despair stage of psychosocial development is which of the following? A. wisdom B. purpose C. care D. will

A

As described by Janis (1982), groupthink is most likely to occur in: A. highly cohesive groups that have a strong directive leader. B. heterogeneous groups that have a transformational leader. C. highly cohesive groups that have a laissez faire leader. D. autonomous groups that don't have a designated leader.

A

As described by Piaget, children in the heteronomous stage of moral reasoning base their judgments about a person's behavior on which of the following? A. the consequences of the behavior B. the person's intentions C. what they would have done in the same situation D. their familiarity with the behavior or situation

A

As described by Wallerstein and Blakeslee (1989), the "sleeper effect" occurs when: A. girls who seem relatively unaffected by the divorce of their parents during childhood exhibit problems in late adolescence or early adulthood. B. boys who exhibit severe problems after their parents' divorce become indistinguishable from boys who exhibit mild problems by the time they reach early adulthood. C. over time, custodial and noncustodial parents become increasingly less consistent in disciplining their children and less responsive to their children's needs. D. the noncustodial parent gradually becomes increasingly less involved in raising his or her children during the second and subsequent years following the divorce.

A

Autonomic hyperactivity, hand tremor, insomnia, transient hallucinations, and generalized tonic-clonic seizures are most suggestive of which of the following? A. alcohol withdrawal B. opioid withdrawal C. stimulant intoxication D. inhalant intoxication

A

Bem's (1981) gender schema theory incorporates elements of which of the following to explain how children learn about gender roles and acquire a gender identity? A. cognitive developmental theory and social learning theory B. cognitive developmental theory and psychodynamic theory C. behavioral learning theory and biological theory D. behavioral learning theory and psychodynamic theory

A

Berscheid's (1991) emotion-in-relationships model identifies which of the following as a cause of strong emotions in close relationships? A. unexpected behaviors that interrupt usual behavioral routines B. coercive interactions that gradually escalate over time C. behaviors that fulfill each partner's most prepotent needs D. internal working models of acceptable and unacceptable behaviors Back to questions

A

Damage to Broca's area produces: A. nonfluent speech with intact comprehension. B. fluent but nonsensical speech with intact comprehension. C. nonfluent speech with severely impaired comprehension. D. fluent but nonsensical speech with impaired comprehension.

A

Dr. Dennis, an organizational psychologist, is hired by a company to develop and validate a new selection test for customer service representatives. When she conducts a concurrent validity study on the test she develops, Dr. Dennis finds that it has a validity coefficient of .65 for men and a validity coefficient of .20 for women. This suggests that the test has: A. differential validity for men and women. B. divergent validity for men and women. C. differential selection for men and women. D. inadequate validity for both men and women.

A

Electrical stimulation of a person's ascending reticular activating system will have which of the following effects? A. A sleeping person will wake up and an awake person will become more alert. B. A sleeping person will have lucid dreams. C. An awake person will engage in unconscious repetitive motor tics. D. An awake person will experience a sense of deja vu.

A

Excessive secretion of the hormone thyroxine by the thyroid gland causes hyperthyroidism, which produces: A. tremor, sweating, tachycardia, difficulty sleeping, anxiety, sensitivity to heat, and unexplained weight loss. B. fatigue, muscle cramps, bradycardia, constipation, dry skin, depression, sensitivity to cold, and unexplained weight gain. C. increased hunger and/or thirst, blurred vision, fatigue, weight loss, and recurrent infections. D. extreme hunger, fatigue, pale skin, tingling lips, dizziness, tremors, irritability, and confusion.

A

For a DSM-5 diagnosis of autism spectrum disorder, symptoms must: A. be present in the early developmental period. B. have persisted for at least six months. C. include an intellectual impairment. D. be related to a known or suspected biological or environmental factor.

A

Functional neuroimaging techniques include all of the following except: A. CT. B. fMRI. C. PET. D. SPECT.

A

In Ainsworth's (Ainsworth et al., 1978) "strange situation," an infant with an ____________ attachment pattern is indifferent to his/her mother and may act as though she's not present, shows little or no distress when she leaves, and ignores her when she returns. Mothers of children with this attachment pattern are often ____________. A. avoidant; rejecting/unresponsive or intrusive/overstimulating B. avoidant; moody/depressed and inconsistent C. ambivalent; rejecting/unresponsive or intrusive/overstimulating D. ambivalent; moody/depressed and inconsistent

A

In a research study, Halloween trick-or-treaters were assigned to one of four conditions: alone and identifiable, alone and unidentifiable, group and identifiable, or group and unidentifiable. For children in each condition, the researcher left the room after instructing them to take only one piece of candy from the bowl on the table. The results indicated that children took the most candy from the bowl when they were in a group and unidentifiable. These results provide support for which of the following? A. deindividuation B. insufficient justification C. behavioral willingness D. groupthink

A

In a research study, children were told not to play with an attractive toy during a free play period. Some of the children were told that the consequence for playing with the toy would be mild punishment, while others were told the consequence would be more severe punishment. All of the children refrained from playing with the toy but, when subsequently asked about the toy, only those who faced the threat of mild punishment said they disliked it. These results are consistent with the predictions of which of the following? A. cognitive dissonance theory B. elaboration likelihood model C. theory of planned behavior D. balance theory

A

Individuals with Wernicke's aphasia have: A. impaired comprehension of spoken and written language and fluent speech that's devoid of meaning. B. impaired comprehension of spoken and written language and nonfluent, labored speech. C. relatively intact comprehension and fluent speech that's devoid of meaning. D. relatively intact comprehension and nonfluent, labored speech.

A

Infants first exhibit some degree of auditory (sound) localization: A. soon after birth. B. at about three to four months of age. C. not until six months of age. D. not until ten months of age.

A

Mowrer's (1960) two-factor theory of learning is most useful for understanding which of the following? A. avoidance conditioning B. escape conditioning C. spontaneous recovery D. operant extinction

A

Myasthenia gravis is an autoimmune disorder that causes degeneration of __________ receptors at neuromuscular junctions, resulting in severe muscle weakness and fatigue. A. acetylcholine B. glutamate C. GABA D. norepinephrine Back to questions

A

Patients with damage to which of the following can be expected to make multiple random errors and perseverative errors on the Wisconsin Card Sorting Test? A. frontal lobe B. parietal lobe C. basal ganglia D. cerebellum

A

People with __________ personality disorder have a lack of desire for social relationships and are uninterested in opportunities to develop close relationships. A. schizoid B. histrionic C. avoidant D. paranoid

A

Research has found that, among lesbian, gay, and bisexual individuals, being open about their sexual orientation to friends and family members is associated with: A. lower levels of anxiety, higher levels of self-esteem, and greater social support. B. lower levels of anxiety and higher levels of self-esteem but lower levels of social support. C. higher levels of anxiety but no effect on self-esteem and social support. D. higher levels of anxiety and lower levels of self-esteem and no effect on social support.

A

Sleepwalking and sleep terror are categorized in the DSM-5 as non-rapid eye movement (NREM) sleep arousal disorders. In contrast to people with nightmare disorder, people with a NREM sleep arousal disorder: A. experience incomplete awakening from sleep after an event and, when awakened, have little or no memory for the event. B. experience incomplete awakening from sleep after an event and, when awakened, have a vivid memory for the event. C. awaken easily and completely after an event and have little or no memory for the event. D. awaken easily and completely after an event and have a vivid memory for the event.

A

Some investigators interested in peer relationships have compared the outcomes for children who are either rejected or neglected by their peers. Their studies have found that: A. rejected children have worse outcomes than neglected children and are less likely to experience a change in status when they change peer groups. B. rejected children have better outcomes than neglected children and are more likely to experience a change in status when they change peer groups. C. rejected and neglected children have similar outcomes, but neglected children are more likely to experience a change in status when they change peer groups. D. rejected and neglected children have similar outcomes, but rejected children are more likely to experience a change in status when they change peer groups.

A

Suicide rates vary somewhat from year to year but, with regard to age and gender, the best conclusion is that, among females, those aged ________ have the highest rate while, among males, those aged ________ have the highest rate. A. 45 to 64; 75 and over B. 15 to 24; 45 to 64 C. 45 to 64; 15 to 24 D. 75 and over; 45 to 64

A

The ____________ is based on the Cattell-Horn-Carroll model of cognitive abilities and provides scores on five factors derived from that model: knowledge, fluid reasoning, quantitative reasoning, visual-spatial processing, and working memory. A. SB5 B. WAIS-IV C. KABC-II D. WJ-IV

A

The course of ____________ varies and may involve an acute onset of symptoms or a stepwise or progressive decline in functioning with fluctuations in symptoms or plateaus of varying length. A. vascular neurocognitive disorder B. alcohol-induced neurocognitive disorder C. neurocognitive disorder due to Alzheimer's disease D. neurocognitive disorder due to Parkinson's disease

A

The factor that distinguishes primary hypertension from secondary hypertension is: A. whether or not the cause of the disorder is known. B. whether or not the disorder responds to medication. C. the age at onset of the disorder. D. the severity of the disorder's symptoms.

A

The organizational development technique known as survey feedback involves obtaining information on which of the following? A. employee attitudes and opinions about important aspects of work B. "critical incidents" associated with safety and productivity problems C. employee skill deficiencies that would benefit from training D. the impact of work-family conflicts on employee productivity and satisfaction

A

The tendency for people to overestimate the frequency of deaths due to plane crashes and shark attacks and underestimate deaths due to heart disease can be attributed to reliance on which of the following? A. availability heuristic B. anchoring and adjustment heuristic C. counterfactual thinking D. false consensus effect

A

The two most commonly used behavioral interventions for ____________ are the squeeze and start/stop techniques. A. premature ejaculation B. erectile dysfunction C. delayed ejaculation D. genito-pelvic pain/penetration disorder

A

To evaluate the test-retest reliability of a newly developed measure of intelligence, a test developer administers the test to the same sample of examinees on two separate occasions. When he correlates the two sets of scores, he obtains a reliability coefficient of .60. To increase this reliability coefficient, the test developer should: A. increase the number of test items and make sure the new sample of examinees is heterogeneous with regard to level of intelligence. B. increase the number of test items and make sure the new sample of examinees is homogeneous with regard to level of intelligence. C. decrease the number of test items and make sure the new sample of examinees is heterogeneous with regard to level of intelligence. D. decrease the number of test items and make sure the new sample of examinees is homogeneous with regard to level of intelligence.

A

Tourette's disorder, ADHD, and obsessive-compulsive disorder have been linked to abnormalities in which of the following? A. basal ganglia B. hypothalamus C. amygdala D. medulla

A

When an examinee's scores on the L, F, and K scales of the MMPI-2 assume a V-shape with a low score on the F scale and high scores on the L and K scales, this suggests which of the following? A. The examinee attempted to make a favorable impression. B. The examinee attempted to fake a mental illness. C. The examinee answered half of the items "true" and the other half "false." D. The examinee answered all of the items "true."

A

When using the Mini Mental State Exam (MMSE) as a screening tool for cognitive impairment in older adults, it's important to keep in mind that use of the standard cutoff score for African American and Latino patients may result in too many ____________ and that __________ the cutoff score for these individuals reduces this problem. A. false positives; lowering B. false positives; raising C. false negatives; lowering D. false negatives; raising

A

Which of the following best describes ethical requirements with regard to pro bono services? A. Because pro bono services are addressed in the General Principles, they're encouraged by the Code. B. Because pro bono services are addressed in the General Principles, they're discouraged by the Code. C. Because pro bono services are addressed in the Ethical Standards, they're required by the Code. D. Because pro bono services are addressed in the Ethical Standards, they're prohibited by the Code.

A

Which of the following best describes sensory memory? A. large capacity and brief duration B. large capacity and long duration C. small capacity and brief duration D. small capacity and long duration

A

Which of the following has NOT been identified as an explanation for bystander apathy? A. social loafing B. pluralistic ignorance C. evaluation apprehension D. diffusion of responsibility

A

Which of the following is based on the assumption that interventions are most effective when they match a client's stage of change? A. transtheoretical model B. health belief model C. reality therapy D. solution-focused therapy

A

Which of the following is the appropriate bivariate correlation coefficient to use when the scores to be correlated are both reported as ranks? A. Spearman B. Pearson C. biserial D. point biserial

A

Which of the following is true about obsessive-compulsive disorder? A. The onset of symptoms is earlier in males and males are more likely to have a comorbid tic disorder. B. The onset of symptoms is earlier in females and females are more likely to have a comorbid tic disorder. C. The onset of symptoms is earlier in males but females are more likely to have a comorbid tic disorder. D. The onset of symptoms is earlier in females but males are more likely to have a comorbid tic disorder.

A

Which of the following provides global scores that correspond to two theoretical models - the Cattell-Horn-Carroll model of cognitive abilities and the Luria neuropsychological processing model? A. Kaufman Assessment Battery for Children, Second Edition B. Slosson Intelligence Test, Revised Third Edition C. Cognitive Assessment System, Second Edition D. Peabody Picture Vocabulary Test, Fourth Edition

A

Wolpe (1958) believed that counterconditioning was responsible for the effectiveness of his technique of systematic desensitization for eliminating an anxiety response to a particular stimulus. However, studies using the dismantling strategy found that its effects are actually due to: A. extinction. B. higher-order conditioning. C. satiation. D. stimulus discrimination.

A

You don't consider the possibility that the rudeness and irritability of a new checkout clerk are due to situational factors but, instead, conclude that he's a very unpleasant person who shouldn't have a job that puts him in contact with the public. Your conclusion illustrates which of the following? A. fundamental attribution error B. ultimate attribution error C. actor-observer effect D. illusory correlation

A

You saw Mr. Miller in therapy for four months when he suddenly terminated therapy with no warning or explanation. In addition, he has not paid for his last five sessions and has not responded to the two letters you've sent him asking him to contact you to discuss his unpaid bill. You want to use a collection agency to collect the money he owes you. To be consistent with ethical requirements, you will: A. notify Mr. Miller that you will contact a collection agency if you don't hear from him by the end of the month. B. notify Mr. Miller of your intent to use a collection agency and explain why you've chosen to do so. C. use a collection agency without notifying Mr. Miller as long as you informed him of this practice during the initial informed consent process. D. contact a collection agency but provide it only with Mr. Miller's name, address, phone number, and the amount owed.

A

You would use which of the following tests to evaluate the receptive vocabulary of a 10-year-old child who has received a diagnosis of autism spectrum disorder? A. PPVT-5 B. Vineland-II C. Leiter-3 D. KABC-II

A

Your initial assessment of a client's symptoms suggest they meet the diagnostic criteria for bipolar II disorder. However, you would eliminate this diagnosis if the client provides you with additional information that indicates that: A. her episodes of depressive symptoms have never met all of the criteria for a major depressive episode. B. her episodes involving an elevated mood have met the criteria for a hypomanic episode but not a manic episode. C. she has had at least two episodes of depressive symptoms but only one episode involving an elevated mood. D. she has had at least two episodes involving an elevated mood but only one episode of depressive symptoms.

A

Your new client is Adam, a 19-year-old college sophomore, who says his restlessness, inability to concentrate, and impulsiveness are interfering with his ability to do well in his classes. He tells you that, when he was a child, his pediatrician thought he had attention-deficit/hyperactivity disorder. However, his symptoms - especially his hyperactivity - started "getting better" when he was in high school. To assign a diagnosis of ADHD to Adam, he must have at least: A. five symptoms of inattention and/or five symptoms of hyperactivity and impulsivity. B. six symptoms of inattention and/or six symptoms of hyperactivity and impulsivity. C. a total of five symptoms of inattention, hyperactivity, and/or impulsivity. D. a total of six symptoms of inattention, hyperactivity, and/or impulsivity.

A

An organizational psychologist would most likely be hired by a company to conduct a job evaluation for the purpose of: A. establishing comparable worth. B. determining the training needs of newly hired employees. C. developing performance appraisal measures. D. writing detailed job descriptions.

A A job evaluation is conducted to facilitate decisions related to compensation. It's often used to establish comparable worth, which is the principle that workers who perform jobs that require the same skills and responsibilities or that are of comparable value to the employer should be paid the same.

When a test has a standard deviation of 10, the test's standard error of measurement will fall between: A. 0 and 10 B. 10 and 1.0 C. 0 and 1.0 D. -1.0 and +1.0

A A test's standard error of measurement equals its standard deviation times the square root of 1 minus the reliability coefficient. A test's reliability coefficient can range from 0 to 1.0, so the standard error of measurement for a test that has a standard deviation of 10 ranges from 0 when the reliability coefficient is 1.0 (10 times the square root of 1 minus 1 equals 0) to 10 when the reliability coefficient is 0 (10 times the square root of 1 minus 0 equals 10).

The elaboration likelihood model (Petty & Cacioppo, 1981) predicts attitude change induced by _______________ is likely to be strong and persistent and produce a change in behavior. A. central route processing B. peripheral route processing C. an alpha processing strategy D. an omega processing strategy

A According to the elaboration likelihood model, the central route involves thoughtful and careful evaluation of the message and is more likely than the peripheral route to produce attitude change that's strong and persistent and can be expected to cause a change in behavior.

The Bender-Gestalt II is a measure of visual-motor integration that's also used as a: A. screening test for brain damage. B. screening test for mental maturity. C. measure of behavioral inhibition. D. measure of executive functioning.

A Although the Bender-Gestalt II does not provide detailed information about the location or effects of brain damage, it's considered useful as a screening test for brain damage, especially right parietal lobe damage.

A therapy client who had an insensitive, authoritarian father begins to respond to her therapist as though he's insensitive and authoritarian. A practitioner of Freudian psychoanalysis would describe this as transference, while a practitioner of Harry Stack Sullivan's interpersonal psychotherapy would view it as: A. a parataxic distortion. B. a boundary disturbance. C. projective identification. D. displacement.

A As described by Sullivan (1953), parataxic distortions occur in all types of interpersonal relationships and involve perceiving others, not on the basis of their actual attributes, but on expectations developed from past interpersonal relationships.

A division of labor and a hierarchy of authority are defining characteristics of which of the following organizational theories? A. Weber's bureaucracy B. McGregor's Theory Y C. Fiedler's contingency theory D. Katz and Kahn's open-system theory

A As described by Weber (1947), an ideal bureaucracy is characterized by a division of labor, a hierarchy of authority, clearly defined rules and procedures, impersonal relationships based on position, and selection and promotion decisions based on an applicant's or employee's technical competence.

Kandel's (2001) research with the sea snail, Aplysia, provided information on neuronal changes associated with: A. learning and memory. B. reproductive behavior. C. goal-oriented behavior. D. pain perception.

A Because of the small number and relatively large size of its neurons, Aplysia has been found to be a useful subject for studying the neuronal changes associated with learning and memory. Kandel (2001), for example, studied classical conditioning in Aplysia and found that it produced changes at existing synapses as well as an increase in the number of synapses.

According to Berry's (1990) acculturation model, a person is ____________ when he or she has rejected his or her own minority culture and the dominant culture. A. marginalized B. separated C. unacculturated D. transmutated

A Berry's model distinguishes between four levels of acculturation that are determined by the person's acceptance or rejection of his/her own minority culture and the majority culture: integrated, assimilated, separated, and marginalized. A person is marginalized when he/she has rejected both cultures.

Therapy based on Boyd-Franklin's (1989) multisystems model: A. combines components of the treatment process with levels at which the therapist can provide treatment. B. includes interventions that target the microsystem, mesosystem, exosystem, macrosystem, and chronosystem. C. combines individual interventions for each family member with group therapy and family therapy. D. involves progressively intervening at the individual, family, and community levels.

A Boyd-Franklin developed the multisystems model specifically for African American families. The model consists of two main axes: Axis I consists of the components of the treatment process (e.g., joining, assessing, restructuring), while Axis II consists of the various levels at which treatment can be applied (e.g., individual, family, nonblood kin, friends, community).

An advocate of leader-member exchange theory is most likely to agree that: A. leaders have qualitatively different relationships with in-group and out-group subordinates. B. leaders have qualitatively different relationships with introverted and extraverted coworkers. C. leaders are most effective when they combine a high degree of consideration with a high degree of initiating structure. D. leaders are most effective when they tailor rewards and other outcomes to each worker's needs.

A Dansereau, Graen, and Haga's (1975) leader-member exchange theory is based on the assumption that leader effectiveness and subordinate outcomes are determined by the nature of the interactions between the leader and the subordinate. It proposes that subordinates are treated as in-group or out-group members based on whether or not the leader perceives them as being competent, trustworthy, and willing to assume responsibility.

One of the questions included in a survey asks employees to rate the fairness of the outcomes of a recent organizational decision regarding raises and bonuses. This question assesses employees' perceptions of which of the following? A. distributive justice B. procedural justice C. normative justice D. interactional justice

A Distributive justice refers to the fairness of the distribution of resources and other outcomes.

Before adding a new selection test to the procedure that's currently being used to make hiring decisions, you would want to make sure that adding the test will increase decision-making accuracy. In other words, you'd want to make sure the new selection test has adequate: A. incremental validity. B. convergent validity. C. differential validity. D. external validity.

A Incremental validity refers to the increase in decision-making accuracy that will occur when a new predictor (e.g., a selection test) is added to the current procedure for making hiring or other types of decisions.

Following a traumatic brain injury, a middle-aged woman experiences considerable anterograde and retrograde amnesia. When she begins to recall events that occurred in the past, she'll most likely remember which of the following first? A. where she and her family went on vacation three years ago. B. what she gave her sister for her birthday last year. C. the argument she had with her oldest daughter three weeks ago. D. the events that happened on the morning of her injury.

A Many patients who experience retrograde amnesia as the result of a traumatic brain injury exhibit "shrinking retrograde amnesia" over time. This means that the period of retrograde amnesia gradually shortens with the most distant memories returning first. Terms of Service | Privacy Policy Copyright © 2022 Academic Licensure and Professional Education Service, LLC

Neurocognitive disorder due to Alzheimer's disease and neurocognitive disorder with Lewy bodies (NCDLB) can be difficult to distinguish, especially in their early stages, but there are differences. Which of the following does NOT accurately describe a difference between the two disorders? A. Motor disturbances are more prominent in early Alzheimer's disease than in early NCDLB. B. Memory loss is usually a more prominent early symptom of early Alzheimer's disease than of early NCDLB. C. Nonvisual hallucinations and systematized delusions are more common in early NCDLB than in early Alzheimer's disease. D. Orthostatic hypotension and other autonomic disturbances are more common in early NCDLB than in early Alzheimer's disease.

A Motor disturbances are a more important cause of disability in the early stages of NCDLB than in the early stages of Alzheimer's disease and include muscle rigidity, tremors, and other motor symptoms associated with Parkinson's disease. Although motor problems are also associated with Alzheimer's disease, they usually do not occur until the disease progresses to the middle or late stage. The other three answers accurately describe differences between the two disorders.

A researcher wants to evaluate the effects of virtual reality exposure for treating the storm, height, and spider phobias of a 34-year-old woman. The best single-subject research design for evaluating this treatment is which of the following? A. multiple baseline B. reversal C. discrete trials D. time series

A Of the three single-subject designs listed in the answers (multiple baseline, reversal, and discrete trials), the multiple baseline design would be the most appropriate because it would allow the researcher to determine if the treatment is effective for any of the woman's phobias by sequentially applying the treatment to them. In addition, the multiple baseline design doesn't require a treatment to be withdrawn once it's been applied to a behavior. Consequently, if the treatment has a beneficial effect on any of the woman's phobias, the researcher would not have to withdraw the treatment during the course of the study just for the sake of assessing its effects.

A therapist instructs a woman who constantly starts arguments with her partner and hates cleaning the house to spend one hour cleaning the house every time she initiates an argument. The therapist is using which of the following techniques? A. an ordeal B. a ritual C. positioning D. prescribing the symptom

A Ordeals are used by strategic family therapists to reduce or eliminate a client's undesirable behavior by instructing the client to do an unpleasant task whenever he or she engages in that behavior. Ideally, the client will stop engaging in the undesirable behavior in order to avoid doing the unpleasant task.

Which of the following interventions is used to prevent the intellectual disabilities caused by phenylketonuria (PKU)? A. a special diet B. surgery C. applied behavior analysis D. sensory integration therapy

A PKU is a recessive gene disorder that causes a lack of the enzyme needed to metabolize phenylalanine. A phenylalanine-restricted diet helps prevent the intellectual and social deficits associated with this disorder.

Soon after a young child learns that the family pet is a "doggie," he starts calling cats and all other furry four-legged animals "doggie." According to Piaget, this is an example of which of the following? A. assimilation B. equilibration C. accommodation D. horizontal decalage

A Piaget proposed that the construction of knowledge involves adaptation, which consists of two processes - assimilation and accommodation. Assimilation occurs when children apply their existing schemes (cognitive structures) to new experiences, while accommodation occurs when children modify their existing schemes to better fit new experiences. Applying the word "doggie" to all furry four-legged animals is an example of assimilation.

The test manual for an academic achievement test indicates that it has an alternate forms reliability coefficient of .80. This means that _____ of variability in test scores is true score variability. A. 80% B. 64% C. 36% D. 20%

A Reliability coefficients are interpreted directly as the percent of variability in test scores that is due to true score variability. When the reliability coefficient is .80, this means that 80% of variability in scores is due to true score variability and 20% is due to measurement error.

Dr. Wong has been hired by a law firm to evaluate a new selection test that will be used to hire legal assistants. To ensure the test has adequate reliability, he will administer the test to a sample of legal assistants to determine if the test: A. produces consistent scores. B. improves decision-making accuracy. C. has adequate face validity. D. is an adequate sample of the relevant knowledge domain.

A Reliability refers to the degree to which a predictor is free from the effects of measurement (random) error and, as a result, provides consistent scores.

Dr. Cho has been seeing Mr. and Mrs. Giovanni in therapy for four months. Mr. Giovanni is an insurance salesman and Mrs. Giovanni works part-time for a residential cleaning service company. Mr. Giovanni is suddenly laid off from his job and they ask if Mrs. Giovanni can exchange home cleaning services for therapy until Mr. Giovanni finds another job. Agreeing to this arrangement would be: A. unethical. B. ethical since it's a temporary arrangement. C. ethical since it involves an exchange of services (rather than goods). D. ethical as long as Dr. Cho discusses the possible conflicts with the couple.

A Standard 6.05 of the APA Ethics Code permits barter in certain circumstances, but this arrangement would not be acceptable because it creates a potentially harmful multiple relationship - i.e., Dr. Cho will be acting as both therapist and employer and, if he's dissatisfied with Mrs. Giovanni's work, this could affect his objectivity in therapy. This answer is also consistent with Standards II.1, II.2, and III.28 of the Canadian Code of Ethics.

According to Alfred Adler, which of the following is the major determinant of whether a person has a healthy or mistaken style of life? A. social interest B. self-efficacy C. object relations D. personal constructs

A Style of life is a central concept in Adler's individual psychology and refers to the ways in which a person attempts to overcome feelings of inferiority and strive for superiority. According to Adler, people whose goals and behaviors reflect a high degree of social interest have a healthy style of life, while those whose goals and behaviors reflect a high degree of self-centeredness rather than social interest have a mistaken style of life.

The analysis of covariance (ANCOVA) is used to: A. statistically remove the effects of an extraneous variable on the dependent variable. B. measure the effects of an extraneous variable on the dependent variable by treating it as an independent variable. C. simultaneously assess the effects of the independent variable on two or more dependent variables. D. simultaneously assess the effects of two or more independent variables on a single dependent variable.

A The ANCOVA is used to statistically remove the effects of an extraneous variable from scores on the dependent variable so that it's easier to detect the effects of the independent variable on the dependent variable. When using the ANCOVA, the extraneous variable is the "covariate."

To use the Taylor-Russell tables to estimate a predictor's incremental validity, you need to know the predictor's criterion-related validity coefficient and which of the following? A. selection ratio and base rate B. selection ratio and number of job openings C. base rate and positive hit rate D. number of job openings and positive hit rate

A The Taylor-Russell tables are used to obtain an estimate of the percent of future employees who will be successful on the job when an employer adds the new predictor to the current selection procedure. To use the Taylor-Russell tables, you need to know the predictor's criterion-related validity coefficient, the selection ratio, and the base rate.

The Wonderlic Personnel Test-Revised is a measure of: A. general mental ability. B. work-related motivation. C. occupational interests. D. work-related integrity.

A The Wonderlic Personnel Test - Revised (WPT-R) is a 12-minute test of general mental ability that's used to predict job success.

Immediately after a training program has been delivered to a group of trainees, the developer of the program conducts a ____________ to assess its outcomes. A. summative evaluation B. formative evaluation C. confirmative evaluation D. meta-evaluation

A The full-scope evaluation model distinguishes between four types of training program evaluation - formative, summative, confirmative, and meta. A summative evaluation is conducted immediately after a training program has been delivered to assess trainee reactions to the program and its effectiveness for meeting its goals.

You don't consider the possibility that the rudeness and irritability of a new checkout clerk are due to situational factors but, instead, conclude that he's a very unpleasant person who shouldn't have a job that puts him in contact with the public. Your conclusion illustrates which of the following? A. fundamental attribution error B. ultimate attribution error C. actor-observer effect D. illusory correlation

A The fundamental attribution error is the tendency to overestimate the role of dispositional factors and underestimate the role of situational factors when making attributions about the behaviors of other people.

The goodness-of-fit model developed by Thomas and Chess (1977) focuses on the match between a child's __________ and the demands of the social environment. A. temperament B. sense of self C. emotional development D. cognitive development

A The goodness-of-fit model proposes that maladjustment in young children is often the result of a mismatch between the child's temperament and the demands of the social environment, especially the parents' expectations and childrearing practices.

An organizational psychologist is hired by the owner of Best Plumbing Supply to determine why so many of her employees seem to have low levels of job motivation and satisfaction. The owner says she recently gave employees salary increases and bonuses for outstanding performance, but this did not have a noticeable effect. Being familiar with ____________, the psychologist will most likely tell the owner that, to increase motivation and satisfaction, she should redesign the employees' jobs so they provide opportunities for autonomy, responsibility, and advancement. A. two-factor theory B. equity theory C. goal-setting theory D. situational leadership theory

A The psychologist's recommendation is most consistent with Herzberg's (1966) two-factor theory which predicts (a) that hygiene factors (e.g., pay, benefits, and work conditions) cause dissatisfaction when they're inadequate but do not contribute to satisfaction or motivation when they're adequate and (b) that motivator factors (e.g., opportunities for autonomy, responsibility, and advancement) do not cause dissatisfaction when they're inadequate but contribute to satisfaction and motivation when they're adequate.

Age-related changes in fluid intelligence have been linked to changes in processing speed and: A. working memory. B. sensory memory. C. recent long-term memory. D. prospective memory.

A The research has consistently found moderate to high correlations between measures of processing speed, working memory, and fluid intelligence. There are several explanations for these correlations. One explanation is that age-related changes in processing speed cause changes in working memory that, in turn, cause changes in fluid intelligence. Another explanation is that age-related changes in processing speed and working memory have independent effects on fluid intelligence.

When using the Mini Mental State Exam (MMSE) as a screening tool for cognitive impairment in older adults, it's important to keep in mind that use of the standard cutoff score for African American and Latino patients may result in too many ____________ and that __________ the cutoff score for these individuals reduces this problem. A. false positives; lowering B. false positives; raising C. false negatives; lowering D. false negatives; raising

A The studies have found that use of the standard cutoff score (24) on the MMSE with African American and Latino patients results in too many false positives. Scores on the MMSE range from 0 to 30, with lower scores indicating greater cognitive impairment, so lowering the cutoff score for these patients reduces this problem.

Vicarious liability refers to which of the following? A. an employer's legal responsibility for the acts of his/her employees B. a chief investigator's responsibility for the welfare of his/her research subjects C. an employer's "psychological contract" with his/her employees D. a therapist's obligation to maintain client confidentiality

A When delegating work to an employee or supervisee, psychologists must be aware that, in certain circumstances, they might have vicarious liability - i.e., they might be legally responsible for the actions of the employee or supervisee.

A 20-year-old who has received a diagnosis of ADHD would likely obtain the highest score on which of the following WAIS-IV Indexes? A. Working Memory B. Verbal Comprehension C. Processing Speed D. Perceptual Reasoning

B

A behavior therapist tells a parent that, to get her son to do his homework, she should reward him with 20 minutes of video game time each time he completes his homework. Then, after the boy is regularly completing his homework, the therapist advises the parent to reward him with 20 minutes of video game time only after he completes his homework three times. The therapist has instructed the parent to begin with a continuous schedule of reinforcement and to then switch to an intermittent schedule, which is referred to as:

B

A client whose symptoms meet the diagnostic criteria for dissociative amnesia is unable to recall any events that occurred for about six weeks after she was brutally raped two years ago. This type of dissociative amnesia is referred to in the DSM-5 as: A. selective. B. localized. C. continuous. D. generalized.

B

A criticism of behaviorally anchored rating scales (BARS) is that: A. they're useful only for managerial-level jobs. B. they're time-consuming to develop. C. they're time-consuming to use when there are many employees to assess. D. they focus on extreme (rather than typical) behaviors.

B

A father is concerned because his son just wants to play video games and never finishes his homework. To ensure his son does his homework, the father tells him he can play video games only after he finishes his homework. The father is using which of the following techniques? A. response cost B. Premack principle C. differential reinforcement D. stimulus control

B

A graduate program in clinical psychology requires first-year students to participate in individual therapy. With regard to the requirements of the APA Ethics Code and the Canadian Code of Ethics, this is: A. acceptable as long as special steps are taken to protect students' confidentiality. B. acceptable as long as there are no multiple relationships between the students and the therapists. C. acceptable only if students are given the choice of participating in an alternative activity. D. unacceptable under any circumstances.

B

A movie production company hires only males for male roles and females for female roles for its action/adventure films, even though this means significantly more males than females are hired. In this situation: A. gender is a business necessity. B. gender is a bona fide occupational qualification. C. the production company's hiring practice is unfair to women. D. the production company's hiring practice is an example of systematic bias.

B

A psychologist is engaged in consultee-centered case consultation when he: A. helps a recently licensed psychotherapist derive a diagnosis and treatment plan for a new client who has symptoms of multiple disorders. B. helps a recently licensed psychotherapist acquire the knowledge and skills she needs to work with clients from a cultural group she has not worked with before. C. works collaboratively with a colleague to develop a relapse prevention program for patients receiving treatment at a community mental health clinic. D. works collaboratively with a team of educational and mental health experts to determine if students qualify for special education.

B

A realistic job preview is used to: A. identify newly hired employees who are likely to benefit from training. B. reduce turnover by ensuring that new employees have accurate job expectations. C. identify the most important job requirements when conducting a job analysis. D. help new employees feel as though they're part of the organization.

B

A supervisor notices that an employee seems to be putting more time and effort into his work but this increase has not been accompanied by an increase in productivity. The employee's behavior is most likely: A. the result of a lack of control over the pace of her work. B. an early sign of job burnout. C. a consequence of chronic work-family conflict. D. a symptom of "survivor syndrome."

B

A young man taking a conventional antipsychotic as a treatment for schizophrenia develops tardive dyskinesia. Of the following, which would be the best course of action in this situation? A. increase the dose of the conventional antipsychotic B. replace the conventional antipsychotic with an atypical antipsychotic C. immediately discontinue the conventional antipsychotic D. have the patient take a low dose of a dopamine antagonist

B

After a skateboarding accident, 17-year-old Cody exhibits clumsiness, slurred speech, and other symptoms associated with alcohol intoxication. Which area of Cody's brain was most likely affected by the accident? A. thalamus B. cerebellum C. reticular formation D. parietal lobe

B

Amato (1993) reviewed empirical research on five factors that have been found to negatively impact the adjustment of children after the divorce of their parents, and he concluded that the research most consistently supports the link between __________ and negative outcomes for children. A. economic hardship B. inter parental conflict C. absence of the noncustodial parent D. adjustment of the custodial parent

B

An early criticism of Carl Jung's conceptualization of __________ was that it's similar to the discredited belief of Jean-Baptiste Lamarck that acquired traits can be inherited. A. individuation B. archetypes C. synchronicity D. prototypes

B

As described by Irvin Yalom (Yalom & Leszcz, 2008), the third formative stage of group therapy is characterized by: A. hesitant participation and search for meaning. B. the development of cohesiveness. C. conflict, dominance, and rebellion. D. the development of counter-dependency.

B

Assuming that a woman's symptoms meet the diagnostic criteria for a diagnosis of delusional disorder, the appropriate type of delusion is erotomanic if she's convinced that: A. her husband is having sexual relations with several of the neighbors. B. a local celebrity she met at a fundraiser six months ago is in love with her. C. every man she meets wants to have sex with her. D. no "decent man" could ever be in love with her.

B

B. F. Skinner (1948) delivered food pellets to pigeons every 15 seconds regardless of their behavior. After a brief period of time, the pigeons began performing unusual behaviors such as bowing, turning, and hopping on one foot. Skinner referred to the pigeons' unusual behaviors as: A. experimental neurosis. B. superstitious behaviors. C. the result of one-trial learning. D. the result of two-factor learning.

B

Before using a selection test to estimate how well job applicants will do on a measure of job performance on their first few days of work, you would want to make sure the selection test has adequate: A. concurrent validity. B. predictive validity. C. differential validity. D. construct validity.

B

By about __________ months of age, children use two words to create a simple sentence - for example, "juice gone," "get toy," and "big doggie." A. 12 to 18 B. 18 to 24 C. 24 to 30 D. 30 to 36

B

Erik Erikson (1978) identified a positive outcome ("virtue") for each of his eight stages of psychosocial development. Which of the following does not accurately match a stage with its positive outcome? A. generativity vs. stagnation: care B. autonomy vs. shame and doubt: purpose C. industry vs. inferiority: competence D. basic trust vs. mistrust: hope

B

Following a presidential election, a person who voted for the winning candidate pays attention to news commentaries that support the new president's actions but ignores commentaries that criticize him. This best illustrates which of the following? A. self-serving bias B. confirmation bias C. illusory correlation D. halo effect

B

Glasser's (1998) reality therapy is based on the premise that ____________ leads to the development of a failure identity. A. a boundary disturbance B. irresponsible behavior C. an unresolved intrapsychic conflict D. incongruence between self and experience

B

In a negatively skewed distribution of scores, the __________ is the lowest score and the __________ is the highest score. A. mode; mean B. mean; mode C. median; mean D. mean; median

B

In response to the "Heinz dilemma," some of Kohlberg's research subjects said it was wrong for Heinz to steal a drug to try to save his dying wife because Heinz would get caught and sent to jail. This response is characteristic of Kohlberg's __________ level of moral development. A. unconventional B. preconventional C. conventional D. postconventional

B

Infants less than three months of age usually begin a sleep period with: A. transitional sleep. B. REM sleep. C. stage 2 sleep. D. stage 3 sleep.

B

John Jr. just turned 14 and has started demanding to be allowed to make his own decisions and to have more privileges and independence from the family. In response, John's parents continue to treat him like a child and have become more punitive in an attempt to keep things the way they were. The parents' response to John's demands illustrates which of the following? A. positive feedback B. negative feedback C. reframing D. restraining

B

Of the Big Five personality traits, _______________ have been found to be most predictive of leader effectiveness. A. openness and conscientiousness B. extraversion and conscientiousness C. agreeableness and emotional stability D. openness and dependability

B

One of the DSM-5 diagnostic criteria for intellectual disability is the presence of deficits in intellectual functioning. As described in the DSM-5, deficits are suggested by a score of about _____ or more standard deviations below the mean on an individualized, standardized intelligence test. A. 1-1/2 B. 2 C. 2-1/2 D. 3

B

Pavlov found that, once dogs were classically conditioned to salivate in response to a tone, they subsequently salivated to tones that were somewhat lower and higher in pitch. He referred to this phenomenon as: A. higher-order conditioning. B. stimulus generalization. C. experimental neurosis. D. response generalization.

B

Practitioners of acceptance and commitment therapy: A. use cognitive restructuring techniques to reduce the frequency and severity of a client's most distressing symptoms. B. teach clients mindfulness techniques that help them perceive symptoms as uncomfortable or harmless transient psychological events. C. use modeling and behavior rehearsal to help clients acquire the interpersonal skills they need to have more satisfying relationships. D. teach clients coping skills that will help them deal more effectively with stressful situations in the future.

B

Presbyopia is an inability to: A. perceive high-frequency sounds. B. focus on close objects. C. distinguish between different tastes. D. distinguish between different colors.

B

Studies investigating the impact of client-therapist match in terms of race and ethnicity suggest that: A. clients tend to prefer therapists of the same race or ethnicity and also have significantly better therapy outcomes when their therapists are of the same race or ethnicity. B. clients tend to prefer therapists of the same race or ethnicity but matching clients and therapists in terms of race and ethnicity has little effect on therapy outcomes. C. clients experience significantly better therapy outcomes when their therapists are of the same race or ethnicity only when they had a strong preference for a therapist of the same race or ethnicity. D. clients experience worse therapy outcomes when their therapists are not of the same race or ethnicity, regardless of their stated preferences.

B

Sue and Sue (2015) propose that a White therapist with an internal locus of control and internal locus of responsibility is likely to encounter the most difficulties when working with clients from an ethnic or racial minority group who have an: A. internal locus of control and internal locus of responsibility. B. internal locus of control and external locus of responsibility. C. external locus of control and internal locus of responsibility. D. external locus of control and external locus of responsibility.

B

The 22-year-old biological son of taller-than-average parents is shorter than average. Which of the following helps explain the son's height? A. genotype B. phenotype C. gene expression D. canalization

B

The Kuder-Richardson Formula 20 (KR-20) can be used to estimate a test's ____________ reliability when test items are scored dichotomously. A. alternate forms B. internal consistency C. test-retest D. inter-rater

B

The WISC-V is appropriate for individuals ages: A. 2.6 to 7.7. B. 6.0 to 16.11. C. 7.6 to 18.11. D. 12.0 to 20.11.

B

The development of which of the following tests was based on Murray's personality theory? A. Myers-Briggs Type Indicator B. Thematic Apperception Test C. Sixteen Personality Factor Questionnaire D. NEO Personality Inventory

B

The first step in Meichenbaum's (1977) self-instructional training is: A. conceptualization. B. cognitive modeling. C. problem identification. D. orientation.

B

The primary goal of Bowen's extended family systems therapy is best described as: A. replacing rigid and diffuse boundaries with clear boundaries. B. increasing the differentiation of family members. C. helping family members take responsibility for their own choices. D. reducing symptoms by disrupting dysfunctional interactions.

B

The research has found that early physical and sexual maturation during adolescence is associated with: A. better outcomes for adolescent girls than for adolescent boys. B. better outcomes for adolescent boys than for adolescent girls. C. better outcomes than late maturation for both boys and girls. D. worse outcomes than late maturation for both boys and girls.

B

The research has found that, in adulthood, __________ memory is most negatively affected by increasing age. A. primary B. secondary C. implicit D. sensory

B

The self-serving bias is the tendency to: A. attribute our successes to controllable factors and our failures to uncontrollable factors. B. attribute our successes to dispositional factors and our failures to situational factors. C. seek out information that confirms our attitudes and beliefs and ignore information that contradicts them. D. bolster our self-esteem by focusing on our successes and minimizing or denying our failures.

B

To assign the DSM-5 diagnosis of intermittent explosive disorder, the individual must be at least _____ years of age or at an equivalent developmental level. A. three B. six C. nine D. twelve

B

To evaluate the inter-rater reliability of a test when scores or ratings on the test represent a nominal scale of measurement, you would use which of the following? A. coefficient alpha B. kappa coefficient C. KR-20 D. Spearman-Brown

B

To help a client with exhibitionistic disorder stop exposing himself in public, a behavior therapist has the client imagine that he's at the mall and is starting to unzip his pants. Once that image is clear, the therapist instructs the client to imagine a police officer running up to him, pushing him up against the wall, putting handcuffs on him, and taking him outside the mall to the police car. The therapist is using which of the following interventions? A. response cost B. covert sensitization C. systematic desensitization D. overcorrection

B

To serve as an expert witness in a court case, a psychologist must be: A. determined to be qualified to do so by the attorney requesting his/her services. B. determined to be qualified to do so by the judge. C. certified as an expert witness by a professional forensic organization. D. certified as an expert witness by the Association of State and Provincial Psychology Boards.

B

When Asch (1956) had subjects write estimates of the length of lines privately immediately after doing so publicly in a group, he found that their conformity dramatically decreased. This provides evidence that the conformity the subjects exhibited when making estimates publicly in the group setting was the result of: A. informational influence. B. normative influence. C. social inhibition. D. social facilitation.

B

When assessing the performance of the employees he supervises, Mr. Stein tends to prioritize an employee's willingness to take responsibility for resolving work-related problems. Consequently, when assessing employees who consistently and enthusiastically assume responsibility, Mr. Stein gives them high ratings on unrelated dimensions of job performance. And when assessing employees who frequently shirk responsibility, Mr. Stein gives them low ratings on all dimensions of job performance. Mr. Stein's tendency is an example of which of the following? A. contrast error B. halo error C. leniency/strictness bias D. similarity bias

B

When designing a training program, providing "identical elements" is most important for: A. maximizing trainee motivation. B. ensuring transfer of training. C. setting realistic training goals. D. retraining older workers.

B

Which of the following combines elements of Bandura's self-efficacy theory, Rogers's person-centered therapy, and the transtheoretical model? A. acceptance and commitment therapy B. motivational interviewing C. interpersonal therapy D. stress inoculation training

B

Which of the following describes the relationship between a test's reliability coefficient and its criterion-related validity coefficient? A. A test's criterion-related validity coefficient can be no greater than its reliability coefficient. B. A test's criterion-related validity coefficient can be no greater than the square root of its reliability coefficient. C. A test's criterion-related validity coefficient can be no greater than the square root of one minus its reliability coefficient. D. A test's criterion-related validity coefficient can be no greater than the square of its reliability coefficient.

B

Which of the following is a type of counterbalanced design? A. Solomon four-group B. Latin square C. factorial D. multiple-baseline

B

Which of the following is considered the most effective way to control extraneous variables? A. random selection of subjects from the population B. random assignment of subjects to the different treatment groups C. statistically removing their effects from the independent variable D. statistically removing their effects from the dependent variable

B

Which of the following is most consistent with ethical guidelines for contingent fees? A. Accepting contingent fees is prohibited under any circumstances. B. Accepting contingent fees should usually be avoided. C. Accepting contingent fees is acceptable when all parties voluntarily agree to this arrangement. D. Accepting contingent fees is acceptable as long as it is not prohibited by law.

B

You are preparing a brochure to advertise your professional services and would like to include testimonials from satisfied clients. To be consistent with ethical requirements, you can solicit testimonials from: A. former (but not current) clients. B. former (but not current) clients as long as they're not vulnerable to undue influence. C. current and former clients as long as they're not compensated for providing them. D. current and former clients as long as they do not include misleading or false information about your services.

B

You have been seeing Evan in therapy for four months to treat his social anxiety. Although Evan seemed to make progress in the first few weeks, his symptoms have not changed since then and you do not think he's benefitting from treatment. When you bring up the possibility of ending therapy and referring him to another therapist, Evan says he feels therapy is very helpful and wants to continue. In terms of ethical requirements, your best course of action would be to: A. continue seeing Evan in therapy since he believes he's benefitting from it. B. continue seeing Evan in therapy if you can agree on specific treatment goals but, if not, set a termination date. C. continue seeing Evan in therapy for several more sessions but, if nothing changes, bring up the possibility of ending therapy again. D. explain to Evan why you believe he's not benefitting from therapy and refer him to another therapist.

B

____________ consists of three overlapping stages: conceptualization, skill acquisition and rehearsal, and application and follow-through. A. Self-instructional training B. Stress inoculation training C. Problem-solving therapy D. Motivational interviewing

B

____________ is a major inhibitory neurotransmitter, and disruption of its production or transmission in certain areas of the brain produces anxiety. A. Acetylcholine B. GABA C. Glutamate D. Dopamine

B

In their Robbers Cave study, Sherif and his colleagues (1966) found that competition between two groups of boys for desired prizes produced a high degree of intergroup hostility. These results provide support for which of the following theories of prejudice and discrimination? A. social identity theory B. realistic conflict theory C. scapegoat theory D. extended contact theory

B According to realistic conflict theory, prejudice and discrimination are the result of direct competition between groups for scarce and valued resources. Evidence for this theory is provided by Sherif's Robbers Cave study in which conflict between groups was generated by having two groups of boys who had strong attachments to members of their own group compete for desired prizes.

Simon's (1957) bounded rationality model of decision-making proposes that the assumptions underlying the rational model are often violated because: A. organizational decisions tend to be incremental. B. decision-makers tend to satisfice rather than optimize. C. the decision-making process is susceptible to groupthink. D. decision-makers rarely get fee

B According to the bounded rationality model, rational decision-making is limited by individual and organizational factors and, as a result, decision makers often satisfice - i.e., they consider alternatives only until a minimally acceptable alternative is found.

The theory of planned behavior (Ajzen & Fishbein, 2005) predicts that a person's behavior intention is affected by all of the following except: A. current attitude. B. need for cognitive constancy. C. subjective norms. D. perceived behavior control.

B According to the theory of planned behavior, attitudes influence a person's behavior intention which, in turn, affects the person's behavior. In addition, three factors influence a person's behavior intention: the person's attitude toward the behavior, what the person thinks others believe he/she should do (subjective norms), and the person's confidence in his/her ability to perform the behavior (perceived behavior control).

A drug that's classified as a(n) __________ of a specific neurotransmitter inhibits the effects of the neurotransmitter. A. agonist B. antagonist C. inverse agonist D. partial agonist

B An antagonist produces no effect on its own but, instead, blocks the action of a neurotransmitter at receptor sites or inhibits the synthesis or release of the neurotransmitter.

A supervisor notices that an employee seems to be putting more time and effort into his work but this increase has not been accompanied by an increase in productivity. The employee's behavior is most likely: A. the result of a lack of control over the pace of her work. B. an early sign of job burnout. C. a consequence of chronic work-family conflict. D. a symptom of "survivor syndrome."

B An early sign of burnout is an increase in the time and effort put into work without an increase in productivity. Other early signs include reduced motivation, increased irritability and negativity, social withdrawal, and somatic symptoms.

When lithium has not reduced the symptoms of mania for an individual who has received a diagnosis of bipolar disorder, an alternative is: A. disulfiram. B. carbamazepine. C. propranolol. D. guanfacine.

B Anticonvulsant drugs are often prescribed for individuals with bipolar disorder who have not responded to lithium or cannot tolerate its side effects. Carbamazepine is one of the anticonvulsant drugs that has been found useful for treating mania.

The questions included in ____________ ask experienced job applicants to describe how they handled specific job-related situations in the past. A. a situational interview B. a behavioral interview C. a case interview D. an informational interview

B Behavioral interviews are based on the assumption that past behavior is the best predictor of future behavior. When conducting a behavioral interview, the interviewer asks job applicants with previous job experience how they responded to specific job-related situations in the past.

Bronfenbrenner's (2004) bioecological theory distinguishes between five environmental systems that impact individual development. According to this theory, the __________ consists of the interactions between elements of the microsystem. A. macrosystem B. mesosystem C. exosystem D. chronosystem

B Bronfenbrenner's bioecological theory distinguishes between five environmental systems: microsystem, mesosystem, exosystem, macrosystem, and chronosystem. The microsystem consists of elements in a child's immediate environment, including the child's family, school, and peers, and the mesosystem consists of interactions between those elements (e.g., between the child's family and school).

Carstensen's (1992) socioemotional selectivity theory proposes that, as we get older: A. emotionally meaningful and knowledge-related goals both become more important. B. emotionally meaningful goals become more important and knowledge-related goals become less important. C. emotionally meaningful goals become less important and knowledge-related goals become more important. D. emotionally meaningful and knowledge-related goals both become less important.

B Carstensen's socioemotional selectivity theory proposes that our goals change as our future time becomes more limited. It distinguishes between two types of goals - knowledge-related and emotionally meaningful - and proposes that, as we get older, we place less importance on knowledge-related goals and more importance on emotionally meaningful goals. In terms of social relationships, this means that younger people prefer relationships that fulfill their knowledge-related goals, while older people prefer relationships that fulfill their emotionally meaningful goals.

Milan systemic family therapists use which of the following to help family members recognize similarities and differences in their perceptions of a family event? A. Socratic questioning B. circular questioning C. ordeals D. rituals

B Circular questioning involves asking each family member the same question. Milan systemic family therapists use circular questioning for several reasons including helping family members identify similarities and differences in their perceptions of and reactions to family events and helping therapists acquire information about family patterns.

Dawis and Lofquist's (1984) theory of work adjustment identifies satisfaction and satisfactoriness as predictors of a person's: A. job choice. B. job tenure. C. work productivity. D. career maturity.

B Dawis and Lofquist's theory of work adjustment identifies tenure as the primary indicator of work adjustment and proposes that it's affected by two factors - satisfaction and satisfactoriness. Satisfaction refers to the employee's satisfaction with the job, while satisfactoriness refers to the employer's satisfaction with the employee.

A newly promoted manager tells you that most of the employees he manages have high levels of motivation but limited skills, and he asks your advice about the best management style in this situation. As an advocate of ______________, you recommend that he adopt a selling leadership style. A. Fiedler's contingency theory B. Hersey and Blanchard's situational leadership theory C. House's path-goal theory D. Vroom-Yetton-Jago's leadership model

B Hersey and Blanchard's (1988) situational leadership theory distinguishes between four leadership styles (telling, selling, participating, delegating) and proposes that the best style depends on a worker's motivation and ability.

The court's ruling in the 1979 case of Larry P. v. Wilson Riles: A. prohibited businesses from using IQ tests to make hiring decisions about African American job applicants. B. prohibited schools from using IQ tests to place African American children in special education classes. C. granted all children with disabilities the right to be provided with public education in the least restrictive environment. D. granted businesses the right to use cognitive ability tests to make employment decisions as long as the tests have adequate validity.

B In the Larry P. case, the court concluded that use of standardized IQ tests to place children in special education classes resulted in a disproportionate number of African American students being placed in those classes and ruled that the tests could no longer be used for that purpose for African American students.

The development of depth perception in infancy involves becoming sensitive to three types of information in a predictable order, with __________ information being the first type that infants are sensitive to. A. interposition B. kinetic C. binocular D. pictorial

B In their study on the development of depth perception, A. Yonas and C. E. Granrud found that infants respond to kinetic cues at one to three months of age, binocular cues at three to five months, and pictorial cues at five to seven months.

Marlatt and Gordon's relapse prevention (RP) model identifies which of the following as the immediate precipitant of alcohol use after a period of abstinence? A. craving for alcohol B. high-risk situations C. lifestyle imbalance D. "apparently irrelevant decisions"

B Marlatt and Gordon's RP model proposes that relapse is the result of immediate determinants and covert antecedents and describes high risk situations (an immediate determinant) as the immediate precipitators of alcohol use after a period of abstinence.

The effects of damage to the left and right hemispheres of the cerebral cortex depend on the specific location of the damage but, with regard to emotions, damage to the left hemisphere is most likely to produce: A. inappropriate indifference or euphoria. B. depression or emotional volatility. C. unprovoked rage and hostility. D. unrealistic confidence and optimism.

B Positive emotions are processed primarily in the left hemisphere, and damage to this hemisphere can produce depression or emotional volatility (which is described by some authors as a catastrophic reaction). Negative emotions are processed primarily in the right hemisphere, and damage can produce inappropriate indifference or euphoria.

During his first session with a family, a structural family therapist notices that, whenever the father tries to shed some light on the family's problems, the mother and teenage son interrupt him and contradict what he's saying. The therapist will describe this interaction as a: A. detouring coalition. B. stable coalition. C. symmetrical interaction. D. complementary interaction.

B Practitioners of Minuchin's structural family therapy distinguish between three rigid family triads - stable coalition, detouring coalition, and triangulation. A stable coalition occurs when two family members (often a parent and child) consistently "gang up" against a third family member (the other parent).

Ella took Spanish classes in her last two years of high school but decided to learn French in college instead of continuing with Spanish. She found that, even after studying French for two semesters, whenever she tried to recall the French equivalent for an English word, she'd recall the Spanish word instead. Ella's experience illustrates which of the following? A. retroactive interference B. proactive interference C. the serial position effect D. the recency effect

B Proactive interference occurs when prior learning (e.g., Spanish) interferes with the ability to learn or recall new information (e.g., French).

Members of quality circles (QCs): A. use statistical analysis, project management, and problem-solving methods to reduce the defect rate of products. B. provide management with recommendations for resolving the work-related problems they've identified. C. have total control over their own work, including task assignments and methods, work schedules, employee selection and training, and performance appraisal. D. identify the unique and best aspects of the organization and generate ideas about how to build on them.

B Quality circles are small groups of employees who volunteer to meet regularly to identify problems and methods for resolving them. The employees then provide management with recommendations for alleviating the identified problems.

A behavior therapist tells a parent that, to get her son to do his homework, she should reward him with 20 minutes of video game time each time he completes his homework. Then, after the boy is regularly completing his homework, the therapist advises the parent to reward him with 20 minutes of video game time only after he completes his homework three times. The therapist has instructed the parent to begin with a continuous schedule of reinforcement and to then switch to an intermittent schedule, which is referred to as: A. shaping. B. thinning. C. fading. D. extinguishing.

B Reducing the amount of positive reinforcement is referred to as thinning - i.e., it involves going from a "thicker" reinforcement schedule to a "thinner" one.

Sherif (1935) used the autokinetic phenomenon to study: A. psychological reactance. B. conformity to group norms. C. compliance with direct requests. D. counterfactual thinking.

B Sherif used the autokinetic phenomenon (an optical illusion in which a stationary point of light appears to move in a dark room) to study conformity to group norms.

You obtain the data you need from a sample of licensed psychologists to calculate a correlation coefficient for their EPPP score and yearly salary five years after taking the exam. If you square the correlation coefficient, you will obtain a(n) _______________, which indicates the amount of variability in yearly salary that's accounted for by EPPP score. A. coefficient of concordance B. coefficient of determination C. kappa coefficient D. eta coefficient

B Squaring a correlation coefficient produces a coefficient of determination, which is a measure of shared variability - or, put another way, a measure of the amount of variability in one variable that is accounted for by variability in another variable.

You would use which of the following to assess the cognitive abilities of a non-English speaking 21-year-old? A. WJ-IV B. Leiter-3 C. SIT-R3-1 D. KABC-II

B The Leiter International Performance Scale, Third Edition (Leiter-3) is a measure of cognitive abilities for individuals ages 3 to 75+. Because of its nonverbal format, it's useful for non-English speakers and individuals with language impairments.

Which of the following is a culture-reduced measure of fluid intelligence? A. Kuhlmann-Anderson B. Raven's Progressive Matrices C. Woodcock-Johnson D. Slosson Intelligence Test

B The Raven's Progressive Matrices (RPM) tests are measures of fluid intelligence and are considered to be culture-reduced because they do not use language and performance does not depend on specific cultural or academic learning. There are three RPM tests: Standard Progressive Matrices, Colored Progressive Matrices, and Advanced Progressive Matrices.

Huntington's disease is a progressive autosomal dominant disorder that has been linked to GABA and glutamate abnormalities in the: A. suprachiasmatic nucleus. B. basal ganglia. C. hypothalamus. D. brainstem.

B The basal ganglia are responsible for voluntary motor control, and the motor symptoms of Huntington's disease have been linked to GABA and glutamate abnormalities in the basal ganglia, especially the caudate nucleus and putamen.

A psychologist is planning a research study to evaluate the effects of a two-hour online lecture on statistics for improving the statistics knowledge of 35 psychologists who have just started studying for the EPPP. All participants will (1) take a pre-test consisting of 50 multiple-choice statistics questions on Monday, (2) attend the online lecture on Wednesday evening, and (3) take a post-test consisting of 50 multiple-choice statistics questions that are equivalent to the pre-test questions on Friday. To analyze the data she obtains in her study, the researcher will use which of the following? A. t-test for a single sample B. t-test for correlated samples C. two-way ANOVA D. single-sample chi-square test

B The first steps in identifying the appropriate statistical test are to identify the independent and dependent variables and the scale of measurement of the dependent variable. This study's independent variable is the lecture on statistics and the dependent variable is statistics test score. The dependent variable is measured on a ratio scale, which means that the statistical test will be used to compare the mean scores obtained by the psychologists on the pre- and post-tests. The t-test and ANOVA are both used to compare mean scores, but because there are only two means, the t-test is the appropriate test. To determine which t-test to use, you determine how the means will be obtained: In this study they will be obtained from a single group of subjects, and the t-test for correlated samples is used when two means are obtained from the same group or from two groups that are related in some way.

____________ preventions are aimed at lowering the prevalence of a disorder by identifying individuals who have early signs or symptoms of the disorder and providing them with an intervention to limit its intensity or duration. A. Primary B. Secondary C. Tertiary D. Quaternary

B The goal of secondary prevention is to stop the progression of a disorder by using a screening test or other procedure to identify individuals with early signs or symptoms of the disorder and providing them with an appropriate intervention.

When assessing the performance of the employees he supervises, Mr. Stein tends to prioritize an employee's willingness to take responsibility for resolving work-related problems. Consequently, when assessing employees who consistently and enthusiastically assume responsibility, Mr. Stein gives them high ratings on unrelated dimensions of job performance. And when assessing employees who frequently shirk responsibility, Mr. Stein gives them low ratings on all dimensions of job performance. Mr. Stein's tendency is an example of which of the following? A. contrast error B. halo error C. leniency/strictness bias D. similarity bias

B The halo error occurs when a rater's rating of an employee on one dimension of job performance affects how the rater rates the employee on all other dimensions.

The item discrimination index (D) provides information on how well a test item discriminates between examinees who obtain a high or low score on the entire test. It ranges in value from: A. 0 to +1.0 B. -1.0 to +1.0. C. 0 to 100 D. -.50 to +.50

B The item discrimination index (D) is calculated by subtracting the percent of examinees in the low scoring group (those who obtained a low score on the entire test) who answered the item correctly from the percent of examinees in the high scoring group (those who obtained a high score on the entire test) who answered the item correctly. D ranges from -1.0 to +1.0 and, the closer D is to 0, the weaker its ability to discriminate.

When people are exposed to arguments that address both sides of a controversial issue, a primacy effect is most likely to occur when: A. one side is presented immediately after the other side and attitudes are measured immediately after the second presentation. B. one side is presented immediately after the other side and attitudes are measured one week later. C. one side is presented one week after the other side and attitudes are measured immediately after the second presentation. D. one side is presented one week after the other side and attitudes are measured one week later.

B The primacy effect is the tendency to remember or be persuaded most by information that's presented first. Miller and Campbell (1959) found that the primacy effect is most likely to occur when the two sides of an argument are presented back-to-back and there's an interval of time between presentation of the second side and attitude assessment.

Based on the results of his research on learning, ____________ concluded that deriving a solution to a problem can be the result of insight, which involves mentally restructuring the elements of the problem in order to see it in a new way. A. Hermann Ebbinghaus B. Wolfgang Kohler C. Edward Thorndike D. Edward Tolman

B The results of Kohler's research with chimpanzees demonstrated that, at least in some situations, problem-solving is not the result of trial-and-error but, instead, sudden insight into the problem - i.e., on having an "aha" experience.

Rates of comorbidity are high for individuals with a diagnosis of schizophrenia. For example, according to the DSM-5, over half of individuals with schizophrenia have which of the following? A. social anxiety disorder B. tobacco use disorder C. obsessive-compulsive disorder D. acute stress disorder

B The studies have found that about 70 to 85% of individuals with schizophrenia are tobacco users and, according to the DSM-5, over half of individuals with this diagnosis meet the diagnostic criteria for tobacco use disorder. Of the disorders listed in the answers, tobacco use disorder is the most common co-occurring disorder for individuals with schizophrenia.

As defined by Wrenn (1985), culturally encapsulated therapists: A. assume that the problems of clients from cultural minority groups are related to their minority status. B. make judgments on the basis of their own cultural assumptions which they view as applicable to all clients regardless of their cultural background. C. recognize the impact of culture on behavior but strive to modify the behavior of clients from cultural minority groups to better fit the demands of the dominant culture. D. attempt to be culturally competent but are unaware of the cultural biases that affect their work with clients from culturally diverse backgrounds.

B This answer is most consistent with Wrenn's definition of cultural encapsulation. According to Wrenn, culturally encapsulated counselors define reality in terms of their own cultural assumptions and assume their assumption are correct and are insensitive to cultural variations.

At the beginning of the school year, a researcher tells a first-grade teacher that a test had been administered to all students in kindergarten and the results indicated that five of her 22 students - Bob G., Liang C., Ellen A., Isadore R., and Lucia V. - will show significant gains in academic achievement this year. Even though the five children were actually randomly selected, all showed significant gains in academic skills by the end of the school year. Which of the following best explains these results? A. the Hawthorne effect B. the Rosenthal effect C. the positive halo effect D. the confirmation bias

B This question describes a study that's similar to the study conducted by Rosenthal and Jacobson (1968) who found that teachers' expectations for students identified as academic "spurters" (but who were actually randomly chosen) had a positive effect on the students' IQ scores and grades. This effect is known by several names including the Rosenthal effect, the Pygmalion effect, and the self-fulfilling prophecy effect

Tiedeman's career decision-making model links vocational identity development to: A. cognitive maturation. B. ego identity development. C. career maturity. D. life roles.

B Tiedeman's career decision-making model views vocational identity development as an ongoing decision-making process that's linked to Erikson's stages of ego identity development.

At a "big box" home improvement store, use of a newly developed selection procedure results in a hiring rate for male salespeople of 50%. In this situation, to be consistent with the 80% (four-fifths) rule, the minimum hiring rate for female salespeople when using this procedure should be _____%. A. 20 B. 40 C. 60 D. 80

B When using the 80% rule, adverse impact is occurring when the hiring rate for a legally protected group is less than 80% of the hiring rate for the majority group. When the hiring rate for male applicants is 50%, the minimum hiring rate for female applicants is 40% (.50 times .80 = .40).

A T-score distribution has a mean of _____ and standard deviation of _____. A. 100; 10 B. 100; 15 C. 50; 10 D. 50; 15

C

A recently divorced psychologist joins an online dating service that matches him to a former therapy client. When deciding what to do in this situation, the psychologist should remember that ethical guidelines: A. prohibit psychologists from becoming sexually involved with former clients under any circumstances. B. prohibit psychologists from becoming sexually involved with former clients only when therapy was terminated less than 12 months ago. C. prohibit psychologists from becoming sexually involved with former clients unless certain conditions are met. D. do not explicitly prohibit psychologists from becoming involved with former therapy clients.

C

A researcher has made a Type II error when she: A. rejects a false null hypothesis. B. rejects a true null hypothesis. C. retains a false null hypothesis. D. retains a true null hypothesis.

C

A stroke involving the middle cerebral artery that affects a patient's dominant hemisphere is most likely to produce which of the following symptoms? A. contralateral hemiparesis, mutism, apathy, confusion, and impaired judgment B. contralateral homonymous hemianopia, unilateral cortical blindness, visual agnosia, and memory loss C. contralateral hemiparesis, contralateral homonymous hemianopia, dysarthria, and aphasia D. contralateral hemiparesis, contralateral homonymous hemianopia, apraxia, and sensory neglect

C

A test developer would use the multitrait-multimethod matrix to evaluate a test's: A. incremental validity. B. criterion-related validity. C. construct validity. D. differential validity.

C

According to Lazarus's (1991) cognitive appraisal theory, __________ appraisal involves considering whether an external event is relevant or irrelevant and, if relevant, whether it's positive/benign or stressful. A. central B. peripheral C. primary D. secondary

C

According to Prochaska and DiClemente's (1983) transtheoretical model, people in the ____________ stage of change are aware of their problem behavior, are weighing the pros and cons of changing, and feel they'll be ready to take the steps necessary to change the behavior within the next six months. A. precontemplation B. preparation C. contemplation D. determination

C

According to gate control theory, pain can be alleviated by: A. taking a placebo. B. taking an SSRI. C. massaging the site of the pain. D. reframing negative thoughts.

C

An adolescent says, "I'm trying real hard to figure out what I want to do with my life and haven't decided whether I want to get married and have children or what college major or career I want to pursue." These comments are most consistent with Marcia's (1966) __________ status. A. diffusion B. foreclosure C. moratorium D. upheaval

C

An advanced sleep phase is most characteristic of: A. adults deprived of REM sleep. B. individuals who are blind. C. older adults. D. infants.

C

As described by Carl Jung, ____________ involves bringing aspects of the personal and collective unconscious into consciousness. A. self-actualization B. introjection C. individuation D. separation-individuation

C

As described by Piaget, the ability to conserve emerges during the concrete operational stage and depends on which of the following? A. transductive reasoning B. hypothetico-deductive reasoning C. transformational thinking D. symbolic thinking

C

Atkinson, Thompson, and Grant's (1993) three-dimensional model of multicultural counseling provides guidelines for choosing the optimal role for therapists when working with clients from ethnic/racial minority groups. For example, it recommends that therapists adopt which of the following roles when a client's level of acculturation is low, the client's problem is external, and the primary goal is remediation? A. consultant B. adviser C. advocate D. change agent

C

Based on her observations of children 2-1/2 to 4 years of age, Mildred Parten (1932) derived six types of social participation that differ in terms of level of social complexity. Of these six types, she identified which of the following as the most socially complex? A. parallel play B. mastery play C. cooperative play D. associative play

C

Chronic __________ excitotoxicity has been linked to Alzheimer's disease, Huntington's disease, and several other neurodegenerative disorders. A. acetylcholine B. serotonin C. glutamate D. GABA

C

Dr. Connor is a solution-focused family therapist, and his new clients are Mr. and Mrs. Bingham and their two sons, 7-year-old Bob and 9-year-old Brett. During the first session, Mrs. Bingham says that their main concern is that Brett's misbehavior causes a lot of problems at home and constantly gets him into trouble at school. After asking Mr. and Mrs. Bingham for some examples of what they mean, Dr. Connor will most likely ask which of the following questions? A. What was happening right before the last time Brett got into trouble? B. Which of Brett's behaviors cause the most problems at home and at school? C. What kind of things does Brett do when he's not causing problems at home? D. What usually happens when Brett gets into trouble at school?

C

Dr. Frank, an organizational psychologist, is hired by a company to develop new performance appraisal measures for maintenance technicians. Dr. Frank's initial step in developing these measures will be to conduct a: A. person analysis. B. needs analysis. C. job analysis. D. job evaluation.

C

Dr. Miller is hired by an insurance company to develop a performance appraisal measure for salespeople. She's most likely to recommend that the company use a relative measure (as opposed to a Likert-type rating scale) because relative measures: A. are more acceptable to both raters and rates. B. allow the rater to focus on typical (rather than maximum) performance. C. are less susceptible to some rater biases. D. provide more useful information for employee feedback.

C

Dr. Oz conducts a study to compare the effects of three treatments (drug, relaxation, and drug plus relaxation) on the systolic blood pressure of patients who have secondary hypertension as the result of three different conditions (tobacco use, chronic alcohol use, or obesity). To analyze the main and interaction effects of treatment and condition on systolic blood pressure, Dr. Oz will use which of the following statistical tests? A. chi-square test for multiple samples B. t-test for independent samples C. two-way ANOVA D. MANOVA

C

Dr. Sipowicz has been seeing Annie for six weeks to treat her social phobia and decides it would benefit Annie if her husband attends several of her sessions. When Dr. Sipowicz discusses this with Annie, she agrees and says she'll bring him to her next session. Annie's husband is seeing another therapist to deal with his gambling problem. Inviting Annie's husband to participate in Annie's therapy sessions is: A. unacceptable since he's already receiving therapy from another therapist. B. unacceptable unless you've determined that he's not vulnerable to undue influence. C. acceptable since you believe his participation in therapy will benefit Annie. D. acceptable as long as he will only participate in a few sessions with Annie.

C

Eight months ago, on the one-year anniversary of the death of her mother due to a brain tumor, 47-year-old Gale began experiencing frequent headaches and dizziness nearly every day. Gale has worried constantly about her health since then and has been spending hours each day searching for information on brain tumors on the Internet. Gale's worrying and time on the Internet has kept her from spending time with her husband, two teen-age daughters, and friends. Gale has had two MRIs that showed no sign of a tumor, and her doctor has been unable to find an explanation for her headaches and dizziness. Based on these symptoms, the most likely diagnosis for Gale is: A. conversion disorder. B. body dysmorphic disorder. C. somatic symptom disorder. D. illness anxiety disorder.

C

Following removal of his medial temporal lobes to treat severe epilepsy, the patient known as H. M. experienced which of the following? A. an inability to recall recent and remote long-term memories B. an inability to recall remote (but not recent) long-term memories C. an inability to form new long-term declarative memories D. an inability to form new procedural memories

C

For a DSM-5 diagnosis of cyclothymic disorder, an adult must have experienced depressive symptoms that do not meet the criteria for a major depressive episode and hypomanic symptoms that do not meet the criteria for a hypomanic episode for at least __________, while a child or adolescent must have experienced symptoms for at least __________. A. 12 months; 6 months B. 18 months; 9 months C. 24 months; 12 months D. 36 months; 18 months

C

For most infants, separation anxiety begins when the infant is between __________ months of age. A. 2 and 4 B. 4 and 6 C. 6 and 8 D. 8 and 10

C

Gaylen, age 19, is brought to therapy by her mother, who says that Gaylen has been "acting weird" lately. She says that Gaylen is normally pretty sensible but that, for the past two months, she's been complaining about hearing voices telling her that she's worthless and a burden on her family and should kill herself. Gaylen's mother also says that, when she tries to have a conversation with Gaylen, it's often impossible to understand what she's saying. When you ask Gaylen about the voices she hears, her response is rambling and incoherent. Assuming that Gaylen's symptoms are not due to another mental disorder, a medical condition, or drug use, the most likely diagnosis is: A. delusional disorder. B. brief psychotic disorder. C. schizophreniform disorder. D. schizophrenia.

C

In a normal distribution of scores, a T-score of _____ is equivalent to a z-score of _____ and a percentile rank of 84. A. 50; 0 B. 50; 1.0 C. 60; 1.0 D. 70; 2.0

C

In a normal distribution, a percentile rank of ____ is one standard deviation above the mean of the distribution. A. 50 B. 68 C. 84 D. 97

C

In a(n) ____________ interaction, one partner adopts a dominant "one-up" position while the other partner adopts a submissive "one-down" position. A. closed B. open C. complementary D. symmetrical

C

In the context of operant conditioning, ________ refers to the systematic and gradual removal of prompts. A. desensitization B. habituation C. fading D. thinning

C

In the context of psychological assessment, the terms "floor" and "ceiling" refer to: A. the lowest and highest true scores an examinee is likely to have, given his or her obtained predictor score. B. the lowest and highest scores an examinee is likely to obtain on a criterion, given his or her predictor score. C. the degree to which a test can discriminate among examinees who have very low levels or very high levels of the characteristic measured by the test. D. the degree to which a test accurately predicts the criterion scores of examinees who obtain very low scores or very high scores on the test.

C

Irvin Yalom (2005) views __________ in group therapy as the equivalent of the therapist-client relationship in individual therapy. A. interpersonal learning B. imitative behavior C. cohesiveness D. universality

C

Kluver and Bucy (1938) found that bilateral lesioning of which of the following areas of the brain in rhesus monkeys caused visual agnosia, reduced fear, increased docility, dietary changes, and abnormal sexual behavior. A. medulla, hippocampus, and frontal lobes B. medulla, hypothalamus, and frontal lobes C. amygdala, hippocampus, and temporal lobes D. amygdala, hypothalamus, and temporal lobes

C

Lesions in the dorsolateral area of the prefrontal cortex are most likely to cause: A. an inability to perform movements that require balance and coordination. B. moderate to severe anterograde and retrograde amnesia. C. deficits in insight, planning, judgment, and other executive functions. D. an inability to recognize familiar objects and people by sight

C

Practitioners of __________ view depression as being the result of unresolved grief, interpersonal deficits, role disputes, or role transitions. A. reality therapy B. personal construct therapy C. interpersonal therapy D. solution-focused therapy

C

Research investigating the effects of age on susceptibility to persuasion suggests that ___________ are most resistant to persuasion. A. young and older adults B. young adults C. middle-aged adults D. older adults

C

Researchers found that the attachment patterns of some young children do not fit the three patterns described by Ainsworth and identified disorganized/disoriented attachment as a fourth pattern that's often characteristic of children who: A. have a below-average level of intellectual ability. B. have a "slow-to-warm up" temperament. C. have been abused or neglected by caregivers. D. had a preterm birth or low birthweight.

C

Samantha, age 16, says that it's not necessary to use protection when having sex with her boyfriend because "there's no way I'm going to get pregnant." According to Elkind (1976), this is an example of which of the following? A. identity moratorium B. magical thinking C. the personal fable D. the self-serving bias

C

Studies investigating the outcomes associated with the compressed workweek have found that it has: A. a similarly strong positive effect on job satisfaction and job productivity. B. a positive effect on job satisfaction but a negative effect on job productivity. C. a stronger positive effect on job satisfaction than on job productivity. D. a stronger positive effect on job productivity than on job satisfaction.

C

Tajfel and Turner's (1986) social identity theory proposes that prejudice and discrimination are attributable to: A. competition between groups for valuable resources. B. displaced hostility onto members of outgroups. C. a natural tendency to categorize people into groups. D. conformity to social norms.

C

The 2015 National Survey on Drug Use and Health (SAMHSA, 2016) found that the largest percentage of respondents ages 12 to 17 used which the following in the previous month? A. tobacco B. cocaine C. alcohol D. marijuana

C

The Oregon model of parent management training developed by Gerald Patterson and his colleagues (1982) was based on their research that found a link between high levels of aggressiveness in children and: A. early attachment insecurity. B. rejecting/neglecting parents. C. coercive family interactions. D. a disorganized home environment.

C

The Solomon four-group design is used to control which of the following threats to a study's internal validity? A. history B. statistical regression C. pretest sensitization D. differential selection

C

The ____________ helps maintain the body's homeostasis by regulating body temperature, fluid and electrolyte balance, blood pressure, and other important functions. A. thalamus B. medulla C. hypothalamus D. caudate nucleus

C

The adoption of a multicultural perspective is characteristic of which stage of Atkinson, Morten, and Sue's (2003) racial/cultural identity development model? A. internalization B. introspection C. integrative awareness D. immersion-emersion

C

The overjustification effect occurs when: A. a person attributes the positive outcomes of his/her behavior to dispositional factors when they were actually due to situational factors. B. a person attributes the negative outcomes of his/her behavior to situational factors when they were actually due to dispositional factors. C. obtaining external rewards for engaging in a behavior reduces intrinsic motivation to perform that behavior. D. obtaining external rewards for engaging in a behavior increases intrinsic motivation to perform that behavior.

C

The results of a factor analysis indicate that a test has a correlation coefficient of .20 with Factor I, .35 with Factor II, and .60 with Factor III. The correlation of .60 indicates that ____% of variability in test scores is explained by Factor III. A. 60 B. 40 C. 36 D. 64

C

To compare an obtained sample mean to a known population mean, you would use which of the following? A. ANCOVA B. one-way ANOVA C. t-test for a single sample D. single sample chi-square test

C

To estimate the effect of shortening or lengthening a test on the test's reliability coefficient, you would use which of the following? A. coefficient of determination B. coefficient alpha C. Spearman-Brown formula D. Kuder-Richardson formula 20

C

When a child learns the word "dada," she initially applies it to all adult males. This is an example of which of the following? A. accommodation B. regularization C. overextension D. overregularization

C

When a patient with schizophrenia is taking a neuroleptic (antipsychotic) drug and develops the symptoms of neuroleptic malignant syndrome, the drug should be: A. gradually decreased until the symptoms stop. B. gradually increased until the symptoms stop. C. discontinued immediately. D. gradually decreased or discontinued immediately depending on the severity of the patient's symptoms.

C

When conducting group therapy, you should: A. remind group members that they're legally required to maintain the confidentiality of other group members. B. remind group members that, because they're in therapy, they're ethically obligated to maintain the confidentiality of other group members. C. discuss the importance of maintaining confidentiality with group members in the initial and subsequent sessions. D. discuss the limits of confidentiality with potential members during the screening interview and have them sign a waiver of confidentiality.

C

When two variables are measured on an interval or ratio scale and their relationship is nonlinear, you would use which of the following correlation coefficients to assess their degree of association? A. Spearman rho B. contingency C. eta D. biserial

C

When using the _____________ heuristic, people judge the likelihood of an event based on the extent to which it resembles a prototype. A. confirmation B. anchoring and adjustment C. representativeness D. availability

C

Which of the following has been found to modify the nature of the relationship between message discrepancy and attitude change? A. the recipient's sense of self-efficacy B. the processing channel used by the recipient of the message C. the credibility of the source of the message D. the level of fear aroused by the message

C

With regard to leadership style, which of the following is the best conclusion that can be drawn about the effects of consideration and initiating structure on job outcomes? A. A high level of consideration is more important than a high level of initiating structure for both satisfaction and performance. B. A high level of initiating structure is more important than a high level of consideration for both satisfaction and performance. C. A high level of consideration is more important for satisfaction, while a high level of initiating structure is more important for performance. D. A high level of consideration is more important for performance, while a high level of initiating structure is more important for satisfaction.

C

You have just received a subpoena requesting that you appear in court to provide information about the diagnosis and treatment of a former client of yours who is suing her current therapist for malpractice. You are unable to contact the client or confirm that she has signed an authorization to release information. In this situation, you should: A. notify the party who issued the subpoena that you cannot appear in court without a signed authorization from the client. B. appear in court as requested but provide only information you believe is relevant to the case. C. appear in court as requested but claim the privilege on behalf of the client. D. appear in court and provide the requested information since privilege is waived in this situation.

C

You receive a letter from a colleague requesting that you send her a copy of the record of Donald D., a former client of yours. The request is accompanied by an authorization to release information signed by Donald. Donald abruptly ended his sessions with you three months ago, at which time he owed you for his past four therapy sessions. You've sent two letters to Donald about his unpaid bill, but he has not responded. To be consistent with ethical requirements, you: A. must forward the record to the colleague since Donald has signed an authorization allowing you to do so. B. may refuse to forward the record to the colleague until you and Donald reach an agreement about how he will pay his outstanding fees. C. may refuse to forward the record to the colleague for nonpayment of fees only if they are not needed for Donald's emergency treatment. D. may refuse to forward the record to the colleague for nonpayment of fees only if you explained this policy to Donald as part of the informed consent process when he began therapy.

C

____________ headaches begin with an aura and involve throbbing pain that may be limited to one side of the head and that worsens with physical activity along with nausea, vomiting, and sensitivity to light and/or sound. A. Sinus B. Cluster C. Classic migraine D. Common migraine

C

____________ was used to develop the Occupational Scales of the Strong Interest Inventory, which means that items included in the scale distinguished between people in the different occupational groups and "people-in-general." A. Rational keying B. Theoretical keying C. Empirical criterion keying D. Homogeneous keying

C

Moscovici's (1980) research on minority influence found that a group member with a minority opinion must rely on which of the following to persuade members with the opposing (majority) opinion to adopt his or her opinion? A. informational influence B. normative influence C. behavioral style D. psychological reactance

C According to Moscovici, a group member with a minority opinion must rely on behavioral style to persuade the majority - i.e., the member must be consistent in expressing his/her opinion without seeming too rigid or dogmatic.

Changing alpha (the level of significance) from .05 to .10: A. increases the probability of making a Type I and a Type II error. B. decreases the probability of making a Type I and a Type II error. C. increases the probability of making a Type I error and decreases the probability of making a Type II error. D. decreases the probability of making a Type I error and increases the probability of making a Type II error.

C Alpha is equal to the probability of making a Type I error, and when alpha is increased from .05 to .10, the probability of making a Type I error increases. And, because there's an inverse relationship between the probability of making a Type I error (rejecting a true null hypothesis) and a Type II error (retaining a false null hypothesis), the probability of making a Type II error decreases when alpha is increased.

Dr. Miller is hired by an insurance company to develop a performance appraisal measure for salespeople. She's most likely to recommend that the company use a relative measure (as opposed to a Likert-type rating scale) because relative measures: A. are more acceptable to both raters and rates. B. allow the rater to focus on typical (rather than maximum) performance. C. are less susceptible to some rater biases. D. provide more useful information for employee feedback.

C An advantage of relative job performance measures is that they reduce leniency, strictness, and central tendency rater biases.

For practitioners of Gestalt therapy, a key contributor to neurotic behavior is which of the following? A. cognitive entanglement B. exposure to conditions of worth C. a contact boundary disturbance D. psychological rigidity

C An assumption underlying Gestalt therapy is that contact boundary disturbances underlie neurosis, and it distinguishes between four main types of boundary disturbance: introjection, retroflection, projection, and confluence.

Your neighbor tells you that one of her co-workers, a licensed psychologist, sometimes makes sexually suggestive comments that make her very uncomfortable and self-conscious. She says she asked him to stop making the comments last week because she finds them offensive, and he said his intention was to be funny but that he'd stop if that's what she wants. In terms of ethical requirements, it appears that the co-worker's comments: A. represent sexual harassment because they're sexual in nature and occurred on more than one occasion. B. represent sexual harassment only if they would be identified as unacceptable by a "reasonable person." C. represent sexual harassment if the co-worker does not stop making sexually suggestive comments after being told they're offensive. D. do not represent sexual harassment because the co-worker was unaware that they were offensive while he was making them.

C As defined in Standard 3.02 of the APA Ethics Code, comments of a sexual nature constitute sexual harassment if (a) they are unwelcome, offensive, or create a hostile workplace and the person knows or is told this or (b) they're "sufficiently severe or intense to be abusive to a reasonable person." The co-worker's comments would meet the first criterion if he continues making them after being told they're unwelcome and offensive. This answer is also consistent with Standard I.4 of the Canadian Code of Ethics which prohibits sexual harassment and with the definition of sexual harassment provided in the Code's Preamble.

In their discussion of the use of culture-specific techniques in psychotherapy, Sue and Zane (1987) emphasize the importance of "gift giving" during the initial therapy sessions when working with Asian American and other non-Western clients. As described by these investigators, gift giving includes: A. establishing credibility by demonstrating expertise. B. ensuring confidentiality. C. instilling hope and providing reassurance. D. introducing the topic of race or ethnicity.

C As described by S. Sue and N. Zane, gift giving involves ensuring that clients feel they have received immediate benefits from therapy and includes normalizing the client's feelings and experiences, providing reassurance, instilling hope, teaching coping skills, and helping the client set goals for therapy. Note that Sue and Zane identify credibility as an important treatment issue when working with non-Western clients, but they describe it as a separate issue and not a factor that contributes to gift giving.

____________ was used to develop the Occupational Scales of the Strong Interest Inventory, which means that items included in the scale distinguished between people in the different occupational groups and "people-in-general." A. Rational keying B. Theoretical keying C. Empirical criterion keying D. Homogeneous keying

C As its name suggests, empirical criterion keying involves including items in a scale or test that distinguish between criterion groups. For the Occupational Scales of the Strong Interest Inventory, the criterion groups were people in the different occupational groups and a group consisting of "people-in-general."

____________ viewed schizophrenia and other severe mental disorders as the result of a multigenerational transmission process. A. Salvador Minuchin B. Jay Haley C. Murray Bowen D. William Glasser

C Bowen considered severe mental disorders to be the result of a multigenerational transmission process in which progressively lower levels of differentiation are transmitted over several generations.

On the MMPI-2, which of the following is suggested by a combination of elevated scores on scales 1 (Hs) and 3 (Hy) and a low score on scale 2 (D)? A. passive-aggressive tendencies B. immaturity and self-centeredness C. somatization of psychological problems D. exaggeration or faking of psychological problems

C Elevated scores on scales 1 (Hs) and 3 (Hy) with a significantly lower score on scale 2 (D) on the MMPI-2 is referred to as the "conversion V" pattern. It's associated with somatization of psychological problems, poor insight into feelings and motivation, and a desire to appear rational and normal.

On average, full siblings and other first-degree relatives share _____ of their genetic material. A. 90% B. 75% C. 50% D. 25%

C Full siblings, fraternal twins, and other first degree relatives share about 50% of their genetic material, while half siblings share about 25%.

Which of the following has been found to modify the nature of the relationship between message discrepancy and attitude change? A. the recipient's sense of self-efficacy B. the processing channel used by the recipient of the message C. the credibility of the source of the message D. the level of fear aroused by the message

C In general, the relationship between amount of attitude change and message discrepancy has an inverted-U shape, with the greatest amount of change being produced by a moderate level of discrepancy between the recipient's attitude and the attitude expressed in the persuasive message. However, when communicator credibility is considered, the relationship for high-credible communicators is linear, with the amount of attitude change increasing as the level of discrepancy increases (Aronson, Turner, & Carlsmith. 1963).

When an occupational interest test provides ipsative scores, this means that an examinee's scores indicate: A. the degree of consistency of his/her interests. B. his/her likelihood of success in different occupations. C. the relative strength of each occupational interest assessed by the test. D. the absolute strength of each occupational interest assessed by the test.

C Ipsative scores are also known as intraindividual scores and provide information on the examinee's relative (rather than absolute) strengths with regard to the interests or other characteristic measured by the test.

An employer is concerned about the low motivation of many of her employees and decides to interview them to obtain information on how to alleviate this problem. Being familiar with Vroom's (1964) expectancy theory, the employer will ask employees about all of the following except: A. expectancy. B. instrumentality. C. commitment. D. valence.

C Knowing that expectancy theory is also known as VIE theory would have helped you identify the correct answer to this question: "V" refers to valence, "I" to instrumentality, and "E" to expectancy.

When conducting a one-way ANOVA, an F-ratio is calculated by dividing the mean square between (MSB) by the mean square within (MSW). The mean square between provides an estimate of variability in dependent variable scores due to: A. treatment effects only. B. error only. C. treatment effects plus error. D. treatment effects minus error.

C MSB is a measure of variability due to a combination of treatment effects plus error, while MSW is a measure of variability due to error only. When MSB is divided by MSW, this produces the F-ratio which provides an estimate of treatment effects.

Mayo (1933) found that, no matter what changes were made to lighting and other physical work conditions, factory workers' productivity increased. They attributed this to the special attention the workers received as research participants and referred to it as the: A. placebo effect. B. confirmation effect. C. Hawthorne effect. D. Barnum effect.

C Mayo and his colleagues conducted a series of studies at the Hawthorne plant of the Western Electric Company to evaluate the effects of physical work conditions on worker productivity. An unexpected finding was that productivity increased regardless of changes in physical conditions and that this was due to the attention workers received as research subjects. Mayo referred to this as the Hawthorne effect.

A company's current selection procedure for computer programmers consists of seven predictors that are used to predict the job performance score that a job applicant will receive six months after being hired. The owner of the company wants to reduce the costs and time required to make selection decisions. Which of the following would be most useful for determining the fewest number of predictors needed to make accurate predictions about applicants' job performance scores? A. linear regression analysis B. discriminant function analysis C. stepwise multiple regression D. factor analysis

C Multiple regression is used to predict a person's score on a single criterion (e.g., job performance measure) using two or more predictors. Stepwise multiple regression is a type of multiple regression that's used to identify the fewest number of predictors needed to make an accurate prediction.

Restitution and positive practice are components of which of the following? A. response cost B. Premack principle C. overcorrection D. differential reinforcement

C Overcorrection is used to reduce or eliminate an undesirable behavior and is classified as a type of positive punishment. It consists of two components: Restitution requires the individual to fix any negative consequences of the behavior (e.g., having a child clean up the food he threw on the floor while the family ate dinner). Positive practice involves practicing more appropriate behaviors (e.g., having the child scrape the uneaten food off the plates of other family members into the garbage can).

Strength, accessibility, and specificity have been identified as factors that affect: A. the extent to which a person can resist persuasion. B. the likelihood that a person's behaviors will alter his/her attitudes. C. the amount of influence a person's attitudes have on his/her behaviors. D. the persuasiveness of an advertisement or other message.

C Research has found that the strength of the relationship between a person's attitudes and his/her behaviors is affected by several factors, including the strength, accessibility, and specificity of the attitudes.

Social facilitation is more likely than social inhibition to occur when: A. the crowd is large. B. the crowd is small. C. the task is easy. D. the task is difficult.

C Social facilitation is an improvement in performance that's caused by the mere presence of other people and affects easy and well-learned tasks.

Studies investigating ____________ often place infants in uncertain situations that include a stranger, a novel toy or other object, or the visual cliff. A. basic temperament B. problem-solving C. social referencing D. self-awareness

C Social referencing refers to the use of the emotional reactions of a parent or other person to determine how to respond in ambiguous situations. Most studies on social referencing in infancy have created an ambiguous situation by presenting the infants with a stranger, a novel object, or the visual cliff while a parent or other caregiver is nearby.

Donald Super's life-career rainbow depicts the relationship between which of the following? A. personality characteristics and characteristics of the work environment B. career anticipation and implementation C. life stages and major life roles D. career maturity and career decisions

C Super created several illustrations to depict the relationships between elements of his life-space, life-span career theory. His life-career rainbow depicts the relationship between life stages and major life roles.

When differentiating between conduct disorder and oppositional defiant disorder, the presence of which of the following suggests that oppositional defiant disorder is the correct diagnosis? A. The child's symptoms do not negatively affect his/her academic, social, or other area of functioning. B. The child's symptoms create conflicts between the child and his/her parents and other adults. C. The child's symptoms include problems related to emotional dysregulation. D. The child's defiant behavior is often performed to gain attention.

C The DSM-5 notes that conduct disorder and oppositional defiant disorder both involve conduct problems and conflicts between the child and his/her parents and other adults. However, the symptoms of oppositional defiant disorder are less severe and the diagnostic criteria include an angry/irritable mood (i.e., emotional dysregulation) which is not included in the criteria for conduct disorder.

Which of the following best describes the premise underlying the James-Lange theory of emotion? A. A stimulus is perceived, the stimulus is interpreted, and bodily arousal, behavior, and emotion then occur simultaneously. B. A stimulus is perceived, bodily arousal and a label for that arousal occur, and behavior and emotion then occur simultaneously. C. A stimulus is perceived, bodily arousal and behavior occur, and emotion is then experienced. D. A stimulus is perceived, the stimulus is interpreted, emotion is experienced, and behavior then occurs.

C The James-Lange theory replaces the idea that "we run because we're afraid" with the idea that "we're afraid because we run." In other words, according to this theory, we perceive a stimulus, we respond to the stimulus with arousal and behavior, and we then experience an emotion as we become aware of our bodily reactions.

Which of the following personality tests provides scores on the "Big Five" personality traits? A. EPPS B. MBTI C. NEO-PI-3 D. 16PF

C The NEO-PI-3 (NEO Personality Inventory-3) provides scores on the five major domains of personality (neuroticism, extraversion, agreeableness, conscientiousness, and openness to experience) and the six facets that contribute to each domain.

You would use the Position Analysis Questionnaire to: A. evaluate the effects of a training program. B. identify the "position power" of managerial-level employees. C. conduct a worker-oriented job analysis. D. conduct an organizational analysis.

C The Position Analysis Questionnaire (PAQ) is a worker-oriented job analysis questionnaire that addresses six categories of work activity: information input, mental processes, work output, relationships with other people, job context, and other characteristics.

The tendency to attribute all problems experienced by students with learning disabilities to their learning disabilities and overlook other possible explanations is an example of the: A. fundamental attribution error. B. halo bias. C. diagnostic overshadowing bias. D. base rate fallacy.

C The diagnostic overshadowing bias is the tendency to attribute all of a person's symptoms to one diagnosis (e.g., to a learning disability) and overlook the possibility that they may be due to a comorbid condition.

Smith, Glass, and Miller (1980) conducted a meta-analysis of 475 psychotherapy outcome studies that compared the outcomes of individuals in treatment groups and no-treatment control groups. Their analysis produced an effect size of .85, which means that the average individual who received therapy was "better off" than about _____% of individuals who needed therapy but did not receive it. A. 50 B. 60 C. 80 D. 90

C The effect size calculated by Smith, Glass, and Miller is a type of standard score that has a mean of 0 and standard deviation of 1.0. This means that an effect size of .85 indicates that the mean score obtained by individuals who received treatment was slightly less than one standard deviation above the mean score obtained by individuals who did not receive treatment. In a normal distribution, 84% of scores fall below the score that is one standard deviation above the mean, so slightly less than 84% of scores fall below a score that is .85 standard deviations above the mean. Therefore, answer C (80%) is the best answer.

The keyword method is most useful for: A. remembering a list of 10 nonsense syllables. B. recalling how to solve a mathematical equation. C. learning the vocabulary of a second language. D. remembering an important event you must attend next week.

C The keyword method involves creating an image that links two words or links a word and its definition and is particularly useful for foreign (second) language learning.

The method of loci is most useful for: A. memorizing a short speech. B. remembering how to pronounce foreign-language words. C. remembering a list of unrelated words. D. linking a list of words to their definitions.

C The method of loci uses a visual image to help encode and recall a list of unrelated words. It involves choosing a familiar location and visually placing each word in a different place in that location.

The owner of several fast-food restaurants tells you she's concerned about the low job satisfaction and poor job performance of many of her employees. She says she's heard that "satisfied workers are productive workers" and asks what the research has found about this. You tell her that the studies support which of the following? A. Satisfied workers are, in fact, always productive workers. B. Productive workers are always satisfied workers. C. Workers who are rewarded for their productivity are most productive and satisfied. D. Workers who are happy with their lives are most productive and satisfied.

C The research has found that the relationship between satisfaction and performance is stronger when pay and other reinforcers are closely linked to performance

The standard error of the mean increases in size as the: A. population standard deviation and sample size decrease. B. population standard deviation and sample size increase. C. population standard deviation increases and sample size decreases. D. population standard deviation decreases and sample size increases.

C The standard error of the mean is the standard deviation of the sampling distribution of the mean and is used to determine how well a sample mean estimates a population mean. It's calculated by dividing the population standard deviation by the square root of the sample size, and it increases as the population standard deviation increases and the sample size decreases, and vice versa.

____________ syndrome affects males, is due to the presence of an extra X chromosome, and causes a number of physical abnormalities. A. Prader-Willi B. Angelman C. Klinefelter D. Turner

C The symptoms of Klinefelter syndrome vary, but most males with an extra X chromosome have undeveloped testes, breast enlargement, long limbs with a short trunk, less facial and body hair, and a low testosterone level.

Job applicants who are hired on the basis of their scores on a job selection test but then obtain unsatisfactory scores on a measure of job performance six months later are: A. false negatives. B. true negatives. C. false positives. D. true positives.

C To identify the correct answer to this question, you have to remember that a person's score on the predictor (in this case, the job selection test) determines whether he/she is a "positive" or "negative" and that the person's score on the criterion (the measure of job performance) determines whether he/she is a "true" or "false" positive or negative. Therefore, for this question, a "true positive," is an applicant who scored above the cutoff on the job selection test and receives satisfactory scores on the job performance measure, while a "false positive" (the correct answer) is an applicant who scored above the cutoff on the job selection test but receives unsatisfactory scores on the job performance measure. A "true negative" is an applicant who scored below the cutoff on the job selection test and would have received unsatisfactory scores on the job performance measure if he/she had been hired, while a "false negative" is an applicant who scored below the cutoff on the job selection test but would have received satisfactory scores on the job performance measure if he/she had been hired.

When considering the optimal method of training, you would most likely choose vestibule training because: A. it's less costly than off-the-job training. B. it's useful for teaching higher-order cognitive skills. C. it's less dangerous than on-the-job training. D. it allows training to be self-administered.

C Vestibule training is a type of off-the-job training that allows trainees to acquire skills using the actual machinery or equipment they'll use on-the-job. It's useful when on-the-job training would be too dangerous.

A patient with severe hemiplegia caused by a lesion in the right side of her brain denies she has any problems moving her left arm and leg. This woman's lack of awareness of her physical impairment is referred to as: A. anomia. B. paresthesia. C. anosognosia. D. prosopagnosia.

C anosognosia is most commonly caused by damage to the right parietal lobe and is characterized by a lack of awareness of one's own physical disability.

Which of the following is most responsible for depth perception of objects that are at a close distance? A. motion parallax B. interposition of objects C. retinal disparity D. linear perspective

C. Retinal Disparity Retinal disparity refers to differences in retinal images in the left and right eyes and is responsible for depth perception of objects that are at a close distance. In contrast, motion parallax (the quicker movement of closer objects across the visual field), interposition (overlap) of objects, and linear perspective are responsible for depth perception of objects that are at a greater distance.

"Collaborative empiricism" best describes the therapeutic relationship in: A. Rogers's person-centered therapy. B. Hayes et al.'s acceptance and commitment therapy. C. Glasser's reality therapy. D. Beck's cognitive behavior therapy.

D

A 41-year-old woman says she feels cold and tired all the time, has been having trouble remembering things, is constipated, has unusually dry skin, and has gained weight even though she's eating less. The woman's symptoms are most suggestive of which of the following endocrine disorders? A. Grave's disease B. Addison's disease C. hypoglycemia D. hypothyroidism

D

A child with dyslexia whose reading problems significantly interfere with his academic performance and have not substantially improved with appropriate intervention would receive a DSM-5 diagnosis of: A. learning disability. B. language disorder. C. reading disorder. D. specific learning disorder.

D

A meta-analysis of the research led Rhodes and Wood (1992) to conclude that _______________ are associated with the greatest susceptibility to persuasion. A. low levels of both self-esteem and intelligence B. moderate levels of both self-esteem and intelligence C. low levels of self-esteem and moderate levels of intelligence D. moderate levels of self-esteem and low levels of intelligence

D

A problem with the paired comparison method of performance assessment is that it: A. focuses on what employees are likely to do rather than what they've actually done. B. focuses on extreme (rather than typical) job-related behaviors. C. is susceptible to the social desirability bias. D. is time-consuming to use when there are many employees to rate.

D

A therapist believes that depression, anxiety, and other disorders are similar for all individuals regardless of their cultural background and that the same treatment approaches are effective for all clients. This therapist has adopted which of the following perspectives? A. autoplastic B. alloplastic C. emic D. etic

D

A(n) __________ therapist is most likely to agree that helping a client achieve awareness of his/her thoughts, feelings, and behaviors in the here-and-now is a primary goal of therapy. A. reality B. Rogerian C. interpersonal D. Gestalt

D

According to Fiedler's (1967) contingency theory, a low LPC leader is most effective in: A. moderately favorable situations. B. very unfavorable situations. C. very favorable situations. D. very unfavorable and very favorable situations.

D

Although ____________ personality disorder usually has a chronic course, its symptoms may become less severe as the person gets older, especially during the fourth decade of life. A. schizoid B. paranoid C. obsessive-compulsive D. antisocial

D

An organizational psychologist familiar with goal-setting theory is likely to tell a supervisor that having supervisees participate in setting their own performance goals will produce goals that are: A. less realistic than the goals the supervisor would set alone. B. the same in terms of difficulty as the goals the supervisor would set alone. C. less difficult than the goals the supervisor would set alone. D. more difficult than the goals the supervisor would set alone.

D

Baddeley's (2000) multi-component model of working memory identifies which of the following as responsible for integrating new verbal and visual information? A. sensory register B. phonological loop C. visuospatial sketchpad D. episodic buffer

D

Based on the results of their research with young rhesus monkeys, Harlow and Zimmerman (1959) concluded that which of the following is most important for the development of attachment? A. familiarity B. provision of nourishment C. a strong emotional bond D. contact comfort

D

Dr. Andersen is having trouble with her abrasive father-in-law, a 58-year-old alcoholic, who moved in with Dr. Andersen and her husband two months ago. She realizes that she hates going home after work and that she's started feeling negatively toward therapy clients who resemble her father-in-law. To be consistent with ethical guidelines, Dr. Andersen should: A. temporarily suspend her practice and seek family therapy with her husband and father-in-law. B. refer all clients who resemble her father-in-law to other mental health professionals. C. realize this is a normal countertransference reaction and closely monitor her behavior with clients who resemble her father-in-law. D. consult with a colleague to help her monitor her effectiveness with clients and determine the best course of action.

D

Dr. Merritt overhears Alice, one of the interns she's supervising, making disparaging remarks about people with addictions to a group of friends on two separate occasions. However, Dr. Merritt has never observed Alice acting in an inappropriate way with clients who have any type of addiction and she hasn't discussed what she overheard with Alice. When Alice receives her performance evaluation, Dr. Merritt has noted that one of the low ratings she assigned was due to her concern that Alice is likely to have trouble working with clients who have an addiction. With regard to ethical guidelines: A. Dr. Merritt's evaluation is acceptable because of the unacceptability of the disparaging remarks Alice made. B. Dr. Merritt's evaluation is acceptable as long as it includes an explanation for her concerns about Alice's ability to work with clients who have an addiction. C. Dr. Merritt's evaluation is acceptable as long as it was provided to Alice in a timely manner and she's willing to discuss her concerns with Alice. D. Dr. Merritt's evaluation is not acceptable because it was not based entirely on Alice's actual performance as an intern.

D

Dr. Osprey conducted a study to evaluate the effects of an anti-drug program on attitudes toward drug use for middle school students from low-income families. Subjects were 662 7th and 8th graders attending an inner-city school. Attitudes toward drugs were assessed in the second week of September, the five-hour anti-drug program was administered in five one-hour sessions during the second week of October, and attitudes toward drugs were then re-assessed in the second week of November. Results indicated that the program significantly increased negative attitudes toward drug use. The biggest threat to the internal validity of this study is which of the following? A. statistical regression B. selection C. reactivity D. history

D

During Dr. Walter's first therapy session with a single mother and her 11-year-old son, the mother says that, since she and her husband divorced six months ago, her son has been hostile and stubborn, doesn't listen to her, and is always getting into trouble at home and at school. Dr. Walter tells the mother to imagine that these problems disappear while she's sleeping that night. He then asks her, "When you wake up tomorrow morning, how will you know that a miracle has happened and that things have changed?" Based on this information, you can conclude that Dr. Walter is a practitioner of which of the following? A. interpersonal therapy B. personal construct therapy C. acceptance and commitment therapy D. solution-focused therapy

D

During process consultation, an organizational development (OD) consultant will most likely focus on which of the following? A. job satisfaction and motivation B. theft, sabotage, and other counterproductive behaviors C. perceptions related to organizational justice D. communication, decision-making, and problem-solving

D

Factor Index scores on the Stanford-Binet Intelligence Scales, Fifth Edition (SB5) have a mean of _____ and standard deviation of _____. A. 10; 3 B. 15; 5 C. 100; 12 D. 100; 15

D

In the context of factor analysis, "oblique" means: A. statistically significant. B. statistically insignificant. C. uncorrelated. D. correlated.

D

In the context of research design, external validity refers to: A. consistency. B. accuracy. C. causality. D. generalizability.

D

In the first phase of one of Pavlov's experiments with dogs, a tone was repeatedly paired with food so that, eventually, a dog salivated when the tone was presented alone. In the second phase of the experiment, a light was repeatedly paired with the tone so that the dog also salivated when the light was presented alone. The procedure used in this experiment is known as: A. stimulus generalization. B. stimulus discrimination. C. delay conditioning. D. higher-order conditioning.

D

Just before you fall asleep, you hear a loud noise in your backyard that scares you, and you immediately jump out of bed, go to the back door, and turn on the outside light. When you look out the window, you find that it's just a raccoon that knocked over the garbage can and you begin to calm down. Which of the following is responsible for returning your body to a calm state? A. enteric nervous system B. somatic nervous system C. sympathetic nervous system D. parasympathetic nervous system

D

Older adults with mild neurocognitive disorder due to Alzheimer's disease would most likely obtain the highest score on the WAIS-IV __________ Index and lowest score on the __________ Index. A. Verbal Comprehension; Working Memory B. Working Memory; Perceptual Reasoning C. Perceptual Reasoning; Processing Speed D. Verbal Comprehension; Processing Speed

D

Parametric statistical tests are more "powerful" than nonparametric tests which means that, when using a parametric test, you're more likely to: A. retain a true null hypothesis. B. reject a true null hypothesis. C. retain a false null hypothesis. D. reject a false null hypothesis.

D

Piaget attributed children's ability to engage in deferred imitation and make-believe play to: A. the process of accommodation. B. the process of equilibration. C. the use of mental operations. D. the ability to create mental representations.

D

Prosopagnosia is the inability to: A. recognize spoken words. B. perceive pain. C. interpret emotions. D. recognize familiar faces.

D

Providing patients who have just completed substance abuse treatment with training in coping and relapse prevention skills is an example of: A. primary prevention. B. secondary prevention. C. tertiary prevention. D. quaternary prevention.

D

Research on the Fagan Test of Intelligence has confirmed that ____________ in infancy is a good predictor of IQ in childhood, adolescence, and young adulthood. A. socio-emotional behavior B. language production and comprehension C. sensorimotor development D. selective attention to novel stimuli

D

Researchers using the mirror task have found that most infants begin to recognize themselves in a mirror by about ____ months of age. A. nine B. twelve C. fifteen D. eighteen

D

Six months after the death of your client, you receive a letter from his adult daughter requesting information about his diagnosis and treatment. To be consistent with ethical requirements, you: A. provide the daughter with the information she has requested since her father is deceased. B. provide the daughter with the information she has requested since her father is deceased and she is a family member. C. provide the daughter with a summary of the information she has requested and offer to discuss the information with her. D. provide the daughter with the information she has requested after obtaining authorization from the client's executor or other legally authorized representative.

D

The DSM-5 diagnosis of gender dysphoria requires marked incongruence between one's experienced/expressed gender and one's assigned gender for at least: A. 12 months for children and six months for adolescents and adults. B. 18 months for children and 12 months for adolescents and adults. C. 12 months for children, adolescents, and adults. D. six months for children, adolescents, and adults.

D

The ____________ acts as a "relay station" for all of the senses except smell. A. pons B. hypothalamus C. hippocampus D. thalamus

D

The concordance rate for schizophrenia for non-twin siblings is: A. 41%. B. 23%. C. 18%. D. 9%.

D

The core components of transformational leadership include all of the following except: A. individualized consideration. B. idealized influence. C. intellectual stimulation. D. group consensus.

D

The most likely DSM-5 diagnosis for a 15-year-old boy who has three motor tics and one vocal tic that started 14 months ago is: A. provisional tic disorder. B. persistent tic disorder. C. Gerstmann's syndrome. D. Tourette's disorder.

D

The technique known as orgasmic reconditioning has been identified as an effective treatment for which of the following? A. delayed ejaculation B. female orgasmic disorder C. premature ejaculation D. paraphilic disorder

D

To be consistent with the requirements of the APA Ethics Code and the Canadian Code of Ethics, the potential limits of confidentiality should ordinarily be discussed with a therapy client: A. as soon as possible. B. as part of the informed consent process. C. at the outset of therapy. D. at the outset of therapy and subsequently as needed.

D

Violation of the psychological contract between employees and their employers has been identified as a contributor to: A. work-family conflict. B. job burnout. C. low organization-based self-esteem. D. downsizing survivor syndrome.

D

What is most likely to happen when Behavior A and Behavior B are being reinforced on the same reinforcement schedule and the reinforcement for Behavior A is suddenly stopped? A. Behaviors A and B will both increase. B. Behaviors A and B will both decrease. C. Behavior A will decrease and Behavior B will stay the same. D. Behavior A will decrease and Behavior B will increase.

D

When using positive reinforcement to establish or increase a behavior, use of which of the following intermittent schedules of reinforcement will maximize the behavior's resistance to extinction? A. fixed interval B. variable interval C. fixed ratio D. variable ratio

D

Which of the following is true about the side effects of the selective serotonin reuptake inhibitors (SSRIs) and the tricyclic antidepressants (TCAs)? A. SSRIs are more likely than TCAs to produce serious cardiovascular effects, while TCAs are more likely to produce anticholinergic effects and sexual dysfunction. B. SSRIs are more likely than TCAs to produce anticholinergic effects and sexual dysfunction, while TCAs are more likely to produce serious cardiovascular effects. C. SSRIs are more likely than TCAs to produce anticholinergic effects, while TCAs are more likely to produce cardiovascular effects and sexual dysfunction. D. SSRIs are more likely than TCAs to produce sexual dysfunction, while TCAs are more likely to produce anticholinergic effects and serious cardiovascular effects.

D

Which of the following uses eye opening, verbal, and motor responses to stimuli to evaluate a patient's level of consciousness? A. Mini Mental State Exam B. Rancho Scale of Cognitive Functioning C. Bender Visual-Motor Gestalt Test D. Glasgow Coma Scale Back to questionsPrevious AnswerNext Answer

D

While walking on a city sidewalk, a disheveled young man suddenly staggers, falls into a doorway, and begins coughing and moaning. He's most likely to get help if: A. there are several male and female bystanders. B. there are five or six male bystanders. B. there are five or six female bystanders. D. there is one bystander.

D

You receive a request from the Ethics Committee to provide it with certain confidential information about a former client of yours who has filed an ethics complaint against her current therapist. You should provide the Committee: A. with the information it has requested. B. only with information you believe will serve the best interests of the client. C. only with information you believe is relevant to the client's complaint against her current therapist. D. with the requested information only after confirming the client has signed an authorization to release information.

D

You would use which of the following to construct a confidence interval around an examinee's predicted criterion score? A. regression equation B. multiple regression equation C. standard error of measurement D. standard error of estimate

D

Your new client's symptoms developed soon after the death of his wife, and you are trying to determine if they meet the criteria for major depressive disorder or uncomplicated bereavement. Which of the following symptoms is more consistent with uncomplicated bereavement? A. The client's feelings of emptiness and sadness have lasted for at least one month. B. The client says he never experienced depressive symptoms before the death of his wife. C. The client's symptoms include an inability to anticipate happiness or pleasure. D. The client's depressive symptoms become worse whenever he thinks about his wife.

D

Organizational analysis, task analysis, person analysis, and demographic analysis are components of which of the following? A. work-oriented job analysis B. worker-oriented job analysis C. job evaluation D. needs analysis

D A needs analysis is conducted to identify training needs and often includes an organizational analysis, task analysis, person analysis, and demographic analysis.

According to Fiedler's (1967) contingency theory, a low LPC leader is most effective in: A. moderately favorable situations. B. very unfavorable situations. C. very favorable situations. D. very unfavorable and very favorable situations.

D According to Fiedler, low LPC leaders are most effective in very unfavorable and very favorable situations, while high LPC leaders are most effective in moderately favorable situations.

As described by Margaret Mahler (1971), _____________ involves four substages: differentiation, practicing, rapprochement, and consolidation. A. the stage of mature attachment B. the stage of autonomy C. deindividuation D. separation-individuation

D As described by Mahler, early development involves a predictable sequence of stages: autistic, symbiotic, and separation-individuation. The separation-individuation stage consists of the four substages listed in the question. By the end of this stage, children have developed the capacity for object constancy which allows them to feel both separate from and connected to significant others.

Atkinson, Morten, and Sue's (2003) racial/cultural identity development model predicts that an African American client in the ____________ stage will most likely prefer having a White therapist. A. disintegration B. resistance C. dissonance D. conformity

D Atkinson, Morten, and Sue's racial/cultural identity development model distinguishes between five stages: conformity, dissonance, resistance and immersion, introspection, and integrative awareness. Minority group members in the conformity stage have negative attitudes toward their own minority group and other minority groups and positive attitudes toward Whites (the dominant group). They view White culture as superior and, with regard to therapy, prefer a White therapist.

When taking a walk after dark with her grandmother, a child says the moon is following them because it wants to help them see where they're going. This is an example of: A. horizontal decalage. B. egocentrism. C. magical thinking. D. animistic thinking.

D Children are engaging in animistic thinking when they believe inanimate objects have thoughts, feelings, and other lifelike qualities. In contrast, magical thinking (answer C) occurs when children erroneously believe they have control over objects or events.

Before using a newly developed 10-item screening test to identify people who are depressed, you administer the test to a sample of clinic patients along with an established (validated) 50-item measure of depression and correlate the two sets of scores. In this situation, you are evaluating the screening test's: A. content validity. B. divergent validity. C. differential validity. D. concurrent validity.

D Concurrent validity is a type of criterion-related validity that involves determining how well a new predictor (e.g., a screening test) estimates current scores on a criterion (e.g., a validated measure of depression).

A psychologist is developing a selection test for a large manufacturing company that will be used to predict the job performance ratings of maintenance technicians six months after they're hired. The test developer will be most interested in ensuring that the test has adequate: A. convergent validity. B. content validity. C. construct validity. D. criterion-related validity.

D Criterion-related validity refers to the degree to which scores on the predictor (selection test) correlate with scores on a criterion (measure of job performance). Before using a predictor to estimate scores on a criterion, it's important to determine if the predictor has adequate criterion-related validity.

You have developed a battery of tests to determine which of five vocational training programs is most appropriate for unemployed young adults who dropped out of high school. Which of the following multivariate techniques will be useful in this situation? A. multiple regression B. multivariate analysis of variance C. canonical correlation D. discriminant function analysis

D Discriminant function analysis is also known as discriminant analysis and is the appropriate technique when two or more predictors will be used to categorize people into one of two or more criterion groups - e.g., to use a person's scores on two or more tests to determine which of five vocational training programs is the best one for him or her.

Damage to certain areas of the dominant parietal lobe can produce Gerstmann's syndrome, which includes all of the following symptoms except: A. finger agnosia. B. agraphia. C. left-right confusion. D. ataxia.

D Gerstmann's syndrome involves four symptoms: finger agnosia, left-right confusion, agraphia (difficulty writing), and acalculia (difficulty performing simple mathematical operations). Even if you're not familiar with Gerstmann's syndrome, you may have been able to identify the correct answer to this question as long as you know that ataxia (loss of muscle control) is usually caused by damage to the cerebellum.

George Kelly (1955) proposed that people develop personal constructs that help them make sense of new information and experiences. As described by Kelly, personal constructs are: A. schemas that distort reality. B. socially acceptable schemas. C. unidimensional. D. bipolar.

D Kelly's personal construct theory proposes that people perceive, interpret, and anticipate events using bipolar constructs - e.g., good/bad, relevant/irrelevant, fair/unfair. It's also based on the assumption that each person has his/her own unique set of personal constructs, and Kelly developed the repertory grid technique to examine an individual's system of personal constructs.

Krumboltz's (1979) social learning theory of career decision-making identifies four factors that influence decision-making. Which of the following is NOT one of the four factors? A. genetic endowment B. environmental conditions and events C. task approach skills D. self-efficacy beliefs

D Krumboltz proposed that career decision-making is affected by the person's genetic endowment and special abilities, environmental conditions and events, learning experiences, and task approach skills.

Long-term potentiation (LTP) has been linked to: A. REM sleep. B. visual processing. C. seizure disorders. D. memory storage.

D LTP refers to an increase in synaptic efficacy as the result of repetitive stimulation and is believed to play an important role in the formation and storage of new memories.

To be consistent with APA guidelines, you would not use which of the following to describe research participants in an article about your study? A. transgender women B. Native American adolescents C. the autism spectrum disorder group D. physically challenged adults

D Of the descriptions given in the answers, "physically challenged adults" is least consistent with guidelines presented in the Publication Manual of the American Psychological Association: Ordinarily, when describing people with a disorder or other condition, people-first language should be used (e.g., adults with physical challenges), but there are exceptions to this general rule. For example, when describing a research study and different groups are distinguished, it would be acceptable to refer to a group as "the autism spectrum disorder group" (answer C).

Privilege refers to: A. the ethical requirement to protect a client's confidentiality. B. the legal requirement to protect a client's right to privacy. C. the legal and ethical requirement to disclose confidential client information only with appropriate authorization to do so. D. the legal requirement to protect client confidentiality in court testimony, depositions, and other legal proceedings.

D Privilege is a legal concept that protects a client's confidential information from being disclosed in legal proceedings.

Which of the following is an example of prospective memory? A. remembering what you did on your last birthday B. remembering what your parents gave you when you graduated from 8th grade C. remembering the definition of a new term you learned while studying for the EPPP D. remembering that you have a dentist appointment next Thursday morning

D Prospective memory is memory for tasks that must be performed or events that will occur in the future.

Which of the following terms is used to describe the tendency of children and adolescents to seek experiences that reinforce their genetic predispositions? A. range of reaction B. canalization C. adaptation D. niche-picking

D S. Scarr and K. McCartney described three types of genetic-environment interactions. One of these is niche-picking, which refers to the tendency of children and adolescents to seek experiences and environments that complement and reinforce their genetic predispositions

In the context of test construction, "shrinkage" is associated with: A. inter-rater reliability. B. factor analysis. C. incremental validity. D. cross-validation.

D Shrinkage is associated with cross-validation and refers to the fact that a validity coefficient is likely to be smaller than the original coefficient when the predictor(s) and criterion are administered to another (cross-validation) sample. Shrinkage occurs because the chance factors that contributed to the relationship between the predictor(s) and criterion in the original sample are not present in the cross-validation sample.

Which of the following is most useful for explaining the results of Schachter's (1959) "misery loves miserable company" study? A. gain-loss theory B. self-verification theory C. equity theory D. social comparison theory

D Social comparison theory has been used to explain the results of Schachter's research. It predicts that, in uncertain situations, people often compare themselves to others to obtain information about themselves.

Francis Galton was surprised when he found that a group of contestants at a country fair produced a more accurate estimate of an oxen's weight when their individual estimates were averaged than the estimate made by any one cattle expert. Averaging the estimates provided by a group of contestants is an example of which of the following tasks? A. disjunctive B. conjunctive C. discretionary D. compensatory

D Steiner (1972) distinguished between five types of group tasks: additive, compensatory, disjunctive, conjunctive, and discretionary. When working on a compensatory task, the group's output is the average of each member's estimate, judgment, or other input.

In the context of factor analysis, "oblique" means: A. statistically significant. B. statistically insignificant. C. uncorrelated. D. correlated.

D The factors extracted (identified) in a factor analysis can be either orthogonal or oblique. Orthogonal factors are uncorrelated, while oblique factors are correlated.

The job characteristics model (Hackman & Oldham, 1980) identifies five core job dimensions that affect worker motivation, satisfaction, and performance. Which of the following is NOT one of these dimensions? A. feedback B. task identity C. autonomy D. work engagement

D The five core job dimensions identified by the job characteristics model are skill variety, task identity, task significance, autonomy, and feedback.

Dr. Haar is concerned that the statistics tests she uses for her introductory statistics class are too difficult since so few students pass them. To make her tests a little easier, she will want to remove some items that have an item difficulty index (p) of ________ and add some items that have an item difficulty index of ________. A. +1.0 and higher; -1.0 and lower B. +.50 and higher; -.50 and lower C. .85 and higher; .15 and lower D. .15 and lower; .85 and higher

D The item difficulty index (p) ranges from 0 to 1.0, with 0 indicating a very difficult item (none of the examinees answered it correctly) and 1.0 indicating a very easy item (all examinees answered it correctly). Therefore, to make the statistics tests easier, Dr. Haar will want to remove some of the very difficult items (e.g., those with a p value of .15 and lower) and add some easy items (e.g., those with a p value of .85 and higher).

The two main types of integrity tests that are used to assist with selection decisions are: A. general and specific trait tests. B. covert and behavior-based tests. C. objective and subjective tests. D. overt and personality-based tests.

D There are two main types of integrity tests: Overt integrity tests ask directly about attitudes toward and previous history of dishonesty and theft, while personality-based integrity tests assess aspects of personality that have been linked to dishonesty, disciplinary problems, sabotage, and other counterproductive behaviors.

Following a stroke, a man shaves only the right side of his face, eats food only on the right side of his plate, and frequently bumps into door frames with the left side of his body. Which of the following areas of the brain was affected by the stroke? A. frontal lobe B. temporal lobe C. occipital lobe D. parietal lobe

D This man's symptoms are characteristic of contralateral neglect, which is also known as hemispatial neglect and is caused by damage to the parietal lobe (most often the right parietal lobe). A person with this disorder is unaware of areas and objects on the side of his/her body opposite to the location of the damage.

Tolman and Honzik (1930) concluded that the rats in their studies formed cognitive maps of a maze as the result of which of the following? A. trial-and-error B. sudden insight C. reinforcement D. latent learning

D Tolman and Honzik found that rats allowed to explore a maze without being given a reward found their way to the goal box once a reward was provided more quickly than did rats that did not explore the maze before being given a reward. Based on these results, they concluded that rats allowed to explore the maze formed cognitive maps of the maze and that this confirmed that latent learning occurs without reinforcement.

A young man complains that "everyone at work hates me." His therapist says it may be true that his coworkers hate him and asks him to list the ways they have expressed their hate. The therapist is using which of the following paradoxical techniques? A. reframing B. prescribing C. restraining D. positioning

D When using positioning, the therapist accepts and exaggerates the client's concern in order to help the client recognize its absurdity or irrationality.


संबंधित स्टडी सेट्स

PHS 112: Physical Science II Test 2

View Set

Rikki-tikki-tavi Study Guide Review, Rikki Tikki Tavi Vocab Combo

View Set

Unit 2 Exam - Demand, Supply, and Prices

View Set

Computer Information Systems Exam 2

View Set

Finance 3150 Business Finance Fall 2018 Chapter 3

View Set

PN Nursing Care of Children Practice B with NGN 2022

View Set